Counting,Proability and Binomial Coefficients












4












$begingroup$


If $$P_{2n+2}=sum_{k=n+2}^{2n+2}{2n+2 choose k}p^kq^{2n+2-k}$$



and,



$$P_{2n}=sum_{k=n+1}^{2n}{2n choose k}p^kq^{2n-k}$$



where $0<p<q<1$ and $q=1-p$



Prove that



$$P_{2n+2}=P_{2n}+{2n choose n}p^{n+2}q^n-{2n choose {n+1}}p^{n+1}q^{n+1}$$



$mathbf {Inspiration:}$ A and B play a series of games where the probability of winning $mathit p$ for A is kept less than 0.5. However A gets to choose in advance the total no. of plays. To win the game one must score more than half the games . If the total no. of games is to be even, How many plays should A choose?



$mathbf {Here}$ $P_{2n}$ and $P_{2n+2}$ represents the probability of A winning the play in $2n$ and $2n+2$ games where $2n$ is considered the optimum number of games










share|cite|improve this question









$endgroup$












  • $begingroup$
    This is perhaps true. It reduces to $binom{2 n}{n+1} left((p-1)^2-, _2F_1left(1,1-n;n+2;frac{p}{p-1}right)right)+(p-1) p binom{2 n}{n}=(p-1) p binom{2 (n+1)}{n+2} , _2F_1left(1,-n;n+3;frac{p}{p-1}right)$. Though I do not know how to prove this.
    $endgroup$
    – ablmf
    Jan 13 at 10:30
















4












$begingroup$


If $$P_{2n+2}=sum_{k=n+2}^{2n+2}{2n+2 choose k}p^kq^{2n+2-k}$$



and,



$$P_{2n}=sum_{k=n+1}^{2n}{2n choose k}p^kq^{2n-k}$$



where $0<p<q<1$ and $q=1-p$



Prove that



$$P_{2n+2}=P_{2n}+{2n choose n}p^{n+2}q^n-{2n choose {n+1}}p^{n+1}q^{n+1}$$



$mathbf {Inspiration:}$ A and B play a series of games where the probability of winning $mathit p$ for A is kept less than 0.5. However A gets to choose in advance the total no. of plays. To win the game one must score more than half the games . If the total no. of games is to be even, How many plays should A choose?



$mathbf {Here}$ $P_{2n}$ and $P_{2n+2}$ represents the probability of A winning the play in $2n$ and $2n+2$ games where $2n$ is considered the optimum number of games










share|cite|improve this question









$endgroup$












  • $begingroup$
    This is perhaps true. It reduces to $binom{2 n}{n+1} left((p-1)^2-, _2F_1left(1,1-n;n+2;frac{p}{p-1}right)right)+(p-1) p binom{2 n}{n}=(p-1) p binom{2 (n+1)}{n+2} , _2F_1left(1,-n;n+3;frac{p}{p-1}right)$. Though I do not know how to prove this.
    $endgroup$
    – ablmf
    Jan 13 at 10:30














4












4








4


0



$begingroup$


If $$P_{2n+2}=sum_{k=n+2}^{2n+2}{2n+2 choose k}p^kq^{2n+2-k}$$



and,



$$P_{2n}=sum_{k=n+1}^{2n}{2n choose k}p^kq^{2n-k}$$



where $0<p<q<1$ and $q=1-p$



Prove that



$$P_{2n+2}=P_{2n}+{2n choose n}p^{n+2}q^n-{2n choose {n+1}}p^{n+1}q^{n+1}$$



$mathbf {Inspiration:}$ A and B play a series of games where the probability of winning $mathit p$ for A is kept less than 0.5. However A gets to choose in advance the total no. of plays. To win the game one must score more than half the games . If the total no. of games is to be even, How many plays should A choose?



$mathbf {Here}$ $P_{2n}$ and $P_{2n+2}$ represents the probability of A winning the play in $2n$ and $2n+2$ games where $2n$ is considered the optimum number of games










share|cite|improve this question









$endgroup$




If $$P_{2n+2}=sum_{k=n+2}^{2n+2}{2n+2 choose k}p^kq^{2n+2-k}$$



and,



$$P_{2n}=sum_{k=n+1}^{2n}{2n choose k}p^kq^{2n-k}$$



where $0<p<q<1$ and $q=1-p$



Prove that



$$P_{2n+2}=P_{2n}+{2n choose n}p^{n+2}q^n-{2n choose {n+1}}p^{n+1}q^{n+1}$$



$mathbf {Inspiration:}$ A and B play a series of games where the probability of winning $mathit p$ for A is kept less than 0.5. However A gets to choose in advance the total no. of plays. To win the game one must score more than half the games . If the total no. of games is to be even, How many plays should A choose?



$mathbf {Here}$ $P_{2n}$ and $P_{2n+2}$ represents the probability of A winning the play in $2n$ and $2n+2$ games where $2n$ is considered the optimum number of games







probability sequences-and-series combinatorics probability-theory binomial-coefficients






share|cite|improve this question













share|cite|improve this question











share|cite|improve this question




share|cite|improve this question










asked Jan 13 at 4:28









Arijit DeyArijit Dey

314




314












  • $begingroup$
    This is perhaps true. It reduces to $binom{2 n}{n+1} left((p-1)^2-, _2F_1left(1,1-n;n+2;frac{p}{p-1}right)right)+(p-1) p binom{2 n}{n}=(p-1) p binom{2 (n+1)}{n+2} , _2F_1left(1,-n;n+3;frac{p}{p-1}right)$. Though I do not know how to prove this.
    $endgroup$
    – ablmf
    Jan 13 at 10:30


















  • $begingroup$
    This is perhaps true. It reduces to $binom{2 n}{n+1} left((p-1)^2-, _2F_1left(1,1-n;n+2;frac{p}{p-1}right)right)+(p-1) p binom{2 n}{n}=(p-1) p binom{2 (n+1)}{n+2} , _2F_1left(1,-n;n+3;frac{p}{p-1}right)$. Though I do not know how to prove this.
    $endgroup$
    – ablmf
    Jan 13 at 10:30
















$begingroup$
This is perhaps true. It reduces to $binom{2 n}{n+1} left((p-1)^2-, _2F_1left(1,1-n;n+2;frac{p}{p-1}right)right)+(p-1) p binom{2 n}{n}=(p-1) p binom{2 (n+1)}{n+2} , _2F_1left(1,-n;n+3;frac{p}{p-1}right)$. Though I do not know how to prove this.
$endgroup$
– ablmf
Jan 13 at 10:30




$begingroup$
This is perhaps true. It reduces to $binom{2 n}{n+1} left((p-1)^2-, _2F_1left(1,1-n;n+2;frac{p}{p-1}right)right)+(p-1) p binom{2 n}{n}=(p-1) p binom{2 (n+1)}{n+2} , _2F_1left(1,-n;n+3;frac{p}{p-1}right)$. Though I do not know how to prove this.
$endgroup$
– ablmf
Jan 13 at 10:30










4 Answers
4






active

oldest

votes


















3












$begingroup$

We consider $(p+q)^2P_{2n}$ instead of $P_{2n}$ (because then our identity will be homogeneous). Then, we will simply compare coefficents of monomials $p^kq^{2n+2-k}$ in both sides of identity.



Note that (from equality $p+q=1$)
$$
P_{2n}=sum_{k=n+1}^{2n}{2n choose k}p^kq^{2n-k}=sum_{k=n+1}^{2n}{2n choose k}p^kq^{2n-k} (p+q)^2,
$$

which equals
$$
sum_{k=n+1}^{2n}{2n choose k}p^kq^{2n-k} (p^2+2pq+q^2)=sum_{k=n+1}^{2n}{2n choose k}p^{k+2}q^{2n-k}+sum_{k=n+1}^{2n}2{2n choose k}p^{k+1}q^{2n-k+1}+sum_{k=n+1}^{2n}{2n choose k}p^{k}q^{2n-k+2}=sum_{k=n+3}^{2n+2}{2n choose k-2}p^{k}q^{2n+2-k}+sum_{k=n+2}^{2n+1}2{2n choose k-1}p^{k}q^{2n+2-k}+sum_{k=n+1}^{2n}{2n choose k}p^{k}q^{2n-k+2}.
$$



Therefore, $P_{2n}$ equals
$$
sum_{k=n+2}^{2n+2}left({2n choose k}+2{2n choose k-1}+{2n choose k-2}right)p^kq^{2n+2-k}-left({2n choose n}p^{n+2}q^{n}-{2n choose n+1}p^{n+1}q^{n+1}right).
$$



We also know that
$$
{2n choose k}+2{2n choose k-1}+{2n choose k-2}=left({2n choose k}+{2n choose k-1}right)+left({2n choose k-1}+{2n choose k-2}right)={2n+1 choose k}+{2n+1 choose k-1}={2n+2 choose k},
$$

so we obtain
$$
P_{2n}=sum_{k=n+2}^{2n+2}{2n+2 choose k}p^kq^{2n+2-k}=P_{2n+2}-left({2n choose n}p^{n+2}q^{n}-{2n choose n+1}p^{n+1}q^{n+1}right).
$$

Hence,
$$
P_{2n+2}=P_{2n}+{2n choose n}p^{n+2}q^{n}-{2n choose n+1}p^{n+1}q^{n+1},
$$

as desired.






share|cite|improve this answer









$endgroup$





















    1












    $begingroup$

    The equality reduces to
    $$
    binom{2 n}{n+1} p^{n+1} (1-p)^{n-1} , _2F_1left(1,1-n;n+2;frac{p}{p-1}right)+binom{2 n}{n} p^{n+2} (1-p)^n-binom{2 n}{n+1} p^{n+1} (1-p)^{n+1}=binom{2 (n+1)}{n+2} (1-p)^{-n+2 (n+1)-2} p^{n+2} , _2F_1left(1,-n;n+3;frac{p}{p-1}right)
    $$

    Let $w=p/(1-p)$, then the above is
    $$
    binom{2 n}{n+1} left((w+1)^2 , _2F_1(1,1-n;n+2;-w)-1right)+w binom{2 n}{n}=w binom{2 (n+1)}{n+2} , _2F_1(1,-n;n+3;-w)
    $$

    We can then extract the coefficient for $w^m$ from both side for $m in {0,dots,n}$ to see that this equality holds.






    share|cite|improve this answer









    $endgroup$





















      1












      $begingroup$

      Forget the requirement that $0<p<q<1$; it is unnecessary. So let $p$ and $q$
      be any reals satisfying $p+q=1$.



      Also, let me extend the stage a little bit:




      Definition. Let $m$ and $n$ be integers such that $ngeq mgeq0$. Then, we
      let
      begin{equation}
      Q_{n,m}=sum_{k=m}^{n}dbinom{n}{k}p^{k}q^{n-k}.
      end{equation}




      With this definition, your $P_{2n}$ is $Q_{2n,n+1}$, while your $P_{2n+2}$ is
      $Q_{2n+2,n+2}$. Thus, your claim becomes:




      Theorem 1. Let $n$ be a nonnegative integer. Then,
      begin{equation}
      Q_{2n+2,n+2}=Q_{2n,n+1}+dbinom{2n}{n}p^{n+2}q^{n}-dbinom{2n}{n+1}
      p^{n+1}q^{n+1}.
      end{equation}




      I shall derive this from the following:




      Lemma 2. Let $m$ and $n$ be integers such that $ngeq mgeq1$. Then,
      begin{equation}
      Q_{n,m}=Q_{n-1,m-1}-dbinom{n-1}{m-1}p^{m-1}q^{n-m+1}.
      end{equation}




      Proof of Lemma 2. From $ngeq mgeq1$, we obtain $n-1geq m-1geq0$. Now,
      the definition of $Q_{n-1,m-1}$ yields
      begin{equation}
      Q_{n-1,m-1}=sum_{k=m-1}^{n-1}dbinom{n-1}{k}p^{k}underbrace{q^{n-1-k}
      }_{=q^{n-k-1}}=sum_{k=m-1}^{n-1}dbinom{n-1}{k}p^{k}q^{n-k-1}.
      label{darij1.pf.l2.0}
      tag{1}
      end{equation}



      But the definition of $Q_{n,m}$ yields
      begin{align}
      Q_{n,m} & =sum_{k=m}^{n}underbrace{dbinom{n}{k}}_{substack{=dbinom
      {n-1}{k-1}+dbinom{n-1}{k}\text{(by the recurrence of the binomial
      coefficients)}}}p^{k}q^{n-k}=sum_{k=m}^{n}left( dbinom{n-1}{k-1}
      +dbinom{n-1}{k}right) p^{k}q^{n-k}nonumber\
      & =sum_{k=m}^{n}dbinom{n-1}{k-1}p^{k}q^{n-k}+sum_{k=m}^{n}dbinom{n-1}
      {k}p^{k}q^{n-k}nonumber\
      & =sum_{k=m-1}^{n-1}underbrace{dbinom{n-1}{left( k+1right) -1}
      }_{=dbinom{n-1}{k}}p^{k+1}underbrace{q^{n-left( k+1right) }}
      _{=q^{n-k-1}}+sum_{k=m}^{n}dbinom{n-1}{k}p^{k}q^{n-k}nonumber\
      & qquadleft( text{here, we have substituted }k+1text{ for }ktext{ in
      the first sum}right) nonumber\
      & =sum_{k=m-1}^{n-1}dbinom{n-1}{k}p^{k+1}q^{n-k-1}+sum_{k=m}^{n}
      dbinom{n-1}{k}p^{k}q^{n-k}.
      label{darij1.pf.l2.1}
      tag{2}
      end{align}



      But we have
      begin{equation}
      sum_{k=m-1}^{n}dbinom{n-1}{k}p^{k}q^{n-k}=sum_{k=m}^{n}dbinom{n-1}{k}
      p^{k}q^{n-k}+dbinom{n-1}{m-1}p^{m-1}q^{n-left( m-1right) }
      end{equation}

      and
      begin{align*}
      sum_{k=m-1}^{n}dbinom{n-1}{k}p^{k}q^{n-k} & =sum_{k=m-1}^{n-1}dbinom
      {n-1}{k}p^{k}q^{n-k}+underbrace{dbinom{n-1}{n}}_{substack{=0\text{(since
      }n-1geq0text{ and }n-1<ntext{)}}}p^{n}q^{n-n}\
      & =sum_{k=m-1}^{n-1}dbinom{n-1}{k}p^{k}q^{n-k}.
      end{align*}

      Comparing these two equalities, we obtain
      begin{equation}
      sum_{k=m}^{n}dbinom{n-1}{k}p^{k}q^{n-k}+dbinom{n-1}{m-1}p^{m-1}q^{n-left(
      m-1right) }=sum_{k=m-1}^{n-1}dbinom{n-1}{k}p^{k}q^{n-k}.
      end{equation}

      Thus,
      begin{equation}
      sum_{k=m}^{n}dbinom{n-1}{k}p^{k}q^{n-k}=sum_{k=m-1}^{n-1}dbinom{n-1}
      {k}p^{k}q^{n-k}-dbinom{n-1}{m-1}p^{m-1}q^{n-left( m-1right) }.
      end{equation}

      Hence, eqref{darij1.pf.l2.1} becomes
      begin{align*}
      Q_{n,m} & =sum_{k=m-1}^{n-1}dbinom{n-1}{k}p^{k+1}q^{n-k-1}+underbrace{sum
      _{k=m}^{n}dbinom{n-1}{k}p^{k}q^{n-k}}_{=sum_{k=m-1}^{n-1}dbinom{n-1}
      {k}p^{k}q^{n-k}-dbinom{n-1}{m-1}p^{m-1}q^{n-left( m-1right) }}\
      & =underbrace{sum_{k=m-1}^{n-1}dbinom{n-1}{k}p^{k+1}q^{n-k-1}+sum
      _{k=m-1}^{n-1}dbinom{n-1}{k}p^{k}q^{n-k}}_{=sum_{k=m-1}^{n-1}dbinom{n-1}
      {k}left( p^{k+1}q^{n-k-1}+p^{k}q^{n-k}right) }-dbinom{n-1}{m-1}
      p^{m-1}underbrace{q^{n-left( m-1right) }}_{=q^{n-m+1}}\
      & =sum_{k=m-1}^{n-1}dbinom{n-1}{k}left( underbrace{p^{k+1}}_{=pp^{k}
      }q^{n-k-1}+p^{k}underbrace{q^{n-k}}_{=qq^{n-k-1}}right) -dbinom{n-1}
      {m-1}p^{m-1}q^{n-m+1}\
      & =sum_{k=m-1}^{n-1}dbinom{n-1}{k}underbrace{left( pp^{k}q^{n-k-1}
      +p^{k}qq^{n-k-1}right) }_{substack{=left( p+qright) p^{k}
      q^{n-k-1}=p^{k}q^{n-k-1}\text{(since }p+q=1text{)}}}-dbinom{n-1}
      {m-1}p^{m-1}q^{n-m+1}\
      & =underbrace{sum_{k=m-1}^{n-1}dbinom{n-1}{k}p^{k}q^{n-k-1}}
      _{substack{=Q_{n-1,m-1}\text{(by eqref{darij1.pf.l2.0})}}}-dbinom
      {n-1}{m-1}p^{m-1}q^{n-m+1}\
      & =Q_{n-1,m-1}-dbinom{n-1}{m-1}p^{m-1}q^{n-m+1}.
      end{align*}

      This proves Lemma 2. $blacksquare$



      Proof of Theorem 1. We have $left( 1-q-pqright) -p^{2}=-left(
      p+1right) underbrace{left( p+q-1right) }_{substack{=0\text{(since
      }p+q=1text{)}}}=0$
      , thus $1-q-pq=p^{2}$.



      Lemma 2 (applied to $2n+2$ and $n+2$ instead of $n$ and $m$) yields
      begin{align}
      Q_{2n+2,n+2} & =underbrace{Q_{2n+1,n+1}}_{substack{=Q_{2n,n}-dbinom{2n}
      {n}p^{n}q^{left( 2n+1right) -left( n+1right) +1}\text{(by Lemma 2,
      applied to }2n+1text{ and }n+1\text{instead of }ntext{ and }mtext{)}
      }}-underbrace{dbinom{2n+1}{n+1}}_{substack{=dbinom{2n}{n}+dbinom{2n}
      {n+1}\text{(by the recurrence of the binomial coefficients)}}}p^{n+1}
      underbrace{q^{left( 2n+2right) -left( n+2right) +1}}_{=q^{n+1}
      }nonumber\
      & =Q_{2n,n}-dbinom{2n}{n}p^{n}underbrace{q^{left( 2n+1right) -left(
      n+1right) +1}}_{=q^{n+1}}-underbrace{left( dbinom{2n}{n}+dbinom
      {2n}{n+1}right) p^{n+1}q^{n+1}}_{=dbinom{2n}{n}p^{n+1}q^{n+1}+dbinom
      {2n}{n+1}p^{n+1}q^{n+1}}nonumber\
      & =Q_{2n,n}-dbinom{2n}{n}p^{n}q^{n+1}-left( dbinom{2n}{n}p^{n+1}
      q^{n+1}+dbinom{2n}{n+1}p^{n+1}q^{n+1}right) nonumber\
      & =Q_{2n,n}-dbinom{2n}{n}p^{n}q^{n+1}-dbinom{2n}{n}p^{n+1}q^{n+1}
      -dbinom{2n}{n+1}p^{n+1}q^{n+1}.
      label{darij1.pf.t1.1}
      tag{3}
      end{align}

      But the definition of $Q_{2n,n+1}$ yields $Q_{2n,n+1}=sum_{k=n+1}^{2n}
      dbinom{2n}{k}p^{k}q^{2n-k}$
      . Meanwhile, the definition of $Q_{2n,n}$ yields
      begin{align*}
      Q_{2n,n} & =sum_{k=n}^{2n}dbinom{2n}{k}p^{k}q^{2n-k}=underbrace{sum
      _{k=n+1}^{2n}dbinom{2n}{k}p^{k}q^{2n-k}}_{=Q_{2n,n+1}}+dbinom{2n}{n}
      p^{n}underbrace{q^{2n-n}}_{=q^{n}}\
      & =Q_{2n,n+1}+dbinom{2n}{n}p^{n}q^{n}.
      end{align*}

      Thus,
      eqref{darij1.pf.t1.1} becomes
      begin{align*}
      Q_{2n+2,n+2} & =underbrace{Q_{2n,n}}_{=Q_{2n,n+1}+dbinom{2n}{n}p^{n}q^{n}
      }-dbinom{2n}{n}p^{n}q^{n+1}-dbinom{2n}{n}p^{n+1}q^{n+1}-dbinom{2n}
      {n+1}p^{n+1}q^{n+1}\
      & =Q_{2n,n+1}+dbinom{2n}{n}p^{n}q^{n}-dbinom{2n}{n}p^{n}q^{n+1}-dbinom
      {2n}{n}p^{n+1}q^{n+1}-dbinom{2n}{n+1}p^{n+1}q^{n+1}\
      & =Q_{2n,n+1}+dbinom{2n}{n}underbrace{left( p^{n}q^{n}-p^{n}
      q^{n+1}-p^{n+1}q^{n+1}right) }_{=left( 1-q-pqright) p^{n}q^{n}}
      -dbinom{2n}{n+1}p^{n+1}q^{n+1}\
      & =Q_{2n,n+1}+dbinom{2n}{n}underbrace{left( 1-q-pqright) }_{=p^{2}}
      p^{n}q^{n}-dbinom{2n}{n+1}p^{n+1}q^{n+1}\
      & =Q_{2n,n+1}+dbinom{2n}{n}underbrace{p^{2}p^{n}}_{=p^{n+2}}q^{n}
      -dbinom{2n}{n+1}p^{n+1}q^{n+1}\
      & =Q_{2n,n+1}+dbinom{2n}{n}p^{n+2}q^{n}-dbinom{2n}{n+1}p^{n+1}q^{n+1}.
      end{align*}

      This proves Theorem 1. $blacksquare$






      share|cite|improve this answer









      $endgroup$





















        0












        $begingroup$

        We have for the first sum



        $$S = sum_{k=0}^n {2n+2choose k+n+2} p^{k+n+2} q^{n-k}$$



        and for the second one



        $$T = sum_{k=0}^{n-1} {2nchoose k+n+1} p^{k+n+1} q^{n-1-k}$$



        and seek to show



        $$S-T =
        {2nchoose n} p^{n+2} q^n - {2nchoose n+1} p^{n+1} q^{n+1}$$



        where $p+q=1.$ We see that with



        $$Q_n = sum_{k=0}^n {2n+2choose k+n+2} p^{k+n+2} q^{n-k}$$



        the claim becomes



        $$Q_n - Q_{n-1} =
        {2nchoose n} p^{n+2} q^n - {2nchoose n+1} p^{n+1} q^{n+1}.$$



        Now



        $$Q_n = p^{n+2} q^n
        sum_{k=0}^n {2n+2choose k+n+2} p^{k} q^{-k}
        = p^{n+2} q^n
        sum_{k=0}^n {2n+2choose n-k} p^{k} q^{-k}
        \ = p^{n+2} q^n
        sum_{k=0}^n p^{k} q^{-k}
        [z^{n-k}] (1+z)^{2n+2}
        = p^{n+2} q^n [z^n] (1+z)^{2n+2}
        sum_{k=0}^n p^{k} q^{-k} z^k.$$



        We may extend $k$ beyond $n$ owing to the coefficient extrator $[z^n]$
        in front:



        $$p^{n+2} q^n [z^n] (1+z)^{2n+2}
        sum_{kge 0} p^{k} q^{-k} z^k
        = p^{n+2} q^n [z^n] (1+z)^{2n+2} frac{1}{1-pz/q}
        \ = p^2 [z^n] (1+pqz)^{2n+2} frac{1}{1-p^2z}.$$



        We thus have



        $$Q_{n-1} = p^2 [z^{n-1}] (1+pqz)^{2n} frac{1}{1-p^2z}
        \ = p^2 [z^n] z (1+pqz)^{2n} frac{1}{1-p^2z}.$$



        Subtracting we find



        $$Q_n - Q_{n-1}
        = p^2 [z^n] ((1+pqz)^2-z) (1+pqz)^{2n} frac{1}{1-p^2z}
        \ = p^2 [z^n] (1-p^2 z) (1-q^2 z)
        (1+pqz)^{2n} frac{1}{1-p^2z}
        \ = p^2 [z^n] (1-q^2 z) (1+pqz)^{2n}
        \ = p^2 [z^n] (1+pqz)^{2n}
        - p^2 [z^n] q^2 z (1+pqz)^{2n}
        \ = p^2 p^n q^n {2nchoose n}
        - p^2 q^2 [z^{n-1}] (1+pqz)^{2n}
        \ = p^{n+2} q^n {2nchoose n}
        - p^2 q^2 p^{n-1} q^{n-1} {2nchoose n-1}.$$



        This is indeed



        $$bbox[5px,border:2px solid #00A000]{
        p^{n+2} q^n {2nchoose n}
        - p^{n+1} q^{n+1} {2nchoose n+1}}$$



        as claimed.



        Remark. It might be simpler not to merge the $p^n q^n$ into the
        coefficient extractor. Further commentary TBA.






        share|cite|improve this answer











        $endgroup$













          Your Answer





          StackExchange.ifUsing("editor", function () {
          return StackExchange.using("mathjaxEditing", function () {
          StackExchange.MarkdownEditor.creationCallbacks.add(function (editor, postfix) {
          StackExchange.mathjaxEditing.prepareWmdForMathJax(editor, postfix, [["$", "$"], ["\\(","\\)"]]);
          });
          });
          }, "mathjax-editing");

          StackExchange.ready(function() {
          var channelOptions = {
          tags: "".split(" "),
          id: "69"
          };
          initTagRenderer("".split(" "), "".split(" "), channelOptions);

          StackExchange.using("externalEditor", function() {
          // Have to fire editor after snippets, if snippets enabled
          if (StackExchange.settings.snippets.snippetsEnabled) {
          StackExchange.using("snippets", function() {
          createEditor();
          });
          }
          else {
          createEditor();
          }
          });

          function createEditor() {
          StackExchange.prepareEditor({
          heartbeatType: 'answer',
          autoActivateHeartbeat: false,
          convertImagesToLinks: true,
          noModals: true,
          showLowRepImageUploadWarning: true,
          reputationToPostImages: 10,
          bindNavPrevention: true,
          postfix: "",
          imageUploader: {
          brandingHtml: "Powered by u003ca class="icon-imgur-white" href="https://imgur.com/"u003eu003c/au003e",
          contentPolicyHtml: "User contributions licensed under u003ca href="https://creativecommons.org/licenses/by-sa/3.0/"u003ecc by-sa 3.0 with attribution requiredu003c/au003e u003ca href="https://stackoverflow.com/legal/content-policy"u003e(content policy)u003c/au003e",
          allowUrls: true
          },
          noCode: true, onDemand: true,
          discardSelector: ".discard-answer"
          ,immediatelyShowMarkdownHelp:true
          });


          }
          });














          draft saved

          draft discarded


















          StackExchange.ready(
          function () {
          StackExchange.openid.initPostLogin('.new-post-login', 'https%3a%2f%2fmath.stackexchange.com%2fquestions%2f3071705%2fcounting-proability-and-binomial-coefficients%23new-answer', 'question_page');
          }
          );

          Post as a guest















          Required, but never shown

























          4 Answers
          4






          active

          oldest

          votes








          4 Answers
          4






          active

          oldest

          votes









          active

          oldest

          votes






          active

          oldest

          votes









          3












          $begingroup$

          We consider $(p+q)^2P_{2n}$ instead of $P_{2n}$ (because then our identity will be homogeneous). Then, we will simply compare coefficents of monomials $p^kq^{2n+2-k}$ in both sides of identity.



          Note that (from equality $p+q=1$)
          $$
          P_{2n}=sum_{k=n+1}^{2n}{2n choose k}p^kq^{2n-k}=sum_{k=n+1}^{2n}{2n choose k}p^kq^{2n-k} (p+q)^2,
          $$

          which equals
          $$
          sum_{k=n+1}^{2n}{2n choose k}p^kq^{2n-k} (p^2+2pq+q^2)=sum_{k=n+1}^{2n}{2n choose k}p^{k+2}q^{2n-k}+sum_{k=n+1}^{2n}2{2n choose k}p^{k+1}q^{2n-k+1}+sum_{k=n+1}^{2n}{2n choose k}p^{k}q^{2n-k+2}=sum_{k=n+3}^{2n+2}{2n choose k-2}p^{k}q^{2n+2-k}+sum_{k=n+2}^{2n+1}2{2n choose k-1}p^{k}q^{2n+2-k}+sum_{k=n+1}^{2n}{2n choose k}p^{k}q^{2n-k+2}.
          $$



          Therefore, $P_{2n}$ equals
          $$
          sum_{k=n+2}^{2n+2}left({2n choose k}+2{2n choose k-1}+{2n choose k-2}right)p^kq^{2n+2-k}-left({2n choose n}p^{n+2}q^{n}-{2n choose n+1}p^{n+1}q^{n+1}right).
          $$



          We also know that
          $$
          {2n choose k}+2{2n choose k-1}+{2n choose k-2}=left({2n choose k}+{2n choose k-1}right)+left({2n choose k-1}+{2n choose k-2}right)={2n+1 choose k}+{2n+1 choose k-1}={2n+2 choose k},
          $$

          so we obtain
          $$
          P_{2n}=sum_{k=n+2}^{2n+2}{2n+2 choose k}p^kq^{2n+2-k}=P_{2n+2}-left({2n choose n}p^{n+2}q^{n}-{2n choose n+1}p^{n+1}q^{n+1}right).
          $$

          Hence,
          $$
          P_{2n+2}=P_{2n}+{2n choose n}p^{n+2}q^{n}-{2n choose n+1}p^{n+1}q^{n+1},
          $$

          as desired.






          share|cite|improve this answer









          $endgroup$


















            3












            $begingroup$

            We consider $(p+q)^2P_{2n}$ instead of $P_{2n}$ (because then our identity will be homogeneous). Then, we will simply compare coefficents of monomials $p^kq^{2n+2-k}$ in both sides of identity.



            Note that (from equality $p+q=1$)
            $$
            P_{2n}=sum_{k=n+1}^{2n}{2n choose k}p^kq^{2n-k}=sum_{k=n+1}^{2n}{2n choose k}p^kq^{2n-k} (p+q)^2,
            $$

            which equals
            $$
            sum_{k=n+1}^{2n}{2n choose k}p^kq^{2n-k} (p^2+2pq+q^2)=sum_{k=n+1}^{2n}{2n choose k}p^{k+2}q^{2n-k}+sum_{k=n+1}^{2n}2{2n choose k}p^{k+1}q^{2n-k+1}+sum_{k=n+1}^{2n}{2n choose k}p^{k}q^{2n-k+2}=sum_{k=n+3}^{2n+2}{2n choose k-2}p^{k}q^{2n+2-k}+sum_{k=n+2}^{2n+1}2{2n choose k-1}p^{k}q^{2n+2-k}+sum_{k=n+1}^{2n}{2n choose k}p^{k}q^{2n-k+2}.
            $$



            Therefore, $P_{2n}$ equals
            $$
            sum_{k=n+2}^{2n+2}left({2n choose k}+2{2n choose k-1}+{2n choose k-2}right)p^kq^{2n+2-k}-left({2n choose n}p^{n+2}q^{n}-{2n choose n+1}p^{n+1}q^{n+1}right).
            $$



            We also know that
            $$
            {2n choose k}+2{2n choose k-1}+{2n choose k-2}=left({2n choose k}+{2n choose k-1}right)+left({2n choose k-1}+{2n choose k-2}right)={2n+1 choose k}+{2n+1 choose k-1}={2n+2 choose k},
            $$

            so we obtain
            $$
            P_{2n}=sum_{k=n+2}^{2n+2}{2n+2 choose k}p^kq^{2n+2-k}=P_{2n+2}-left({2n choose n}p^{n+2}q^{n}-{2n choose n+1}p^{n+1}q^{n+1}right).
            $$

            Hence,
            $$
            P_{2n+2}=P_{2n}+{2n choose n}p^{n+2}q^{n}-{2n choose n+1}p^{n+1}q^{n+1},
            $$

            as desired.






            share|cite|improve this answer









            $endgroup$
















              3












              3








              3





              $begingroup$

              We consider $(p+q)^2P_{2n}$ instead of $P_{2n}$ (because then our identity will be homogeneous). Then, we will simply compare coefficents of monomials $p^kq^{2n+2-k}$ in both sides of identity.



              Note that (from equality $p+q=1$)
              $$
              P_{2n}=sum_{k=n+1}^{2n}{2n choose k}p^kq^{2n-k}=sum_{k=n+1}^{2n}{2n choose k}p^kq^{2n-k} (p+q)^2,
              $$

              which equals
              $$
              sum_{k=n+1}^{2n}{2n choose k}p^kq^{2n-k} (p^2+2pq+q^2)=sum_{k=n+1}^{2n}{2n choose k}p^{k+2}q^{2n-k}+sum_{k=n+1}^{2n}2{2n choose k}p^{k+1}q^{2n-k+1}+sum_{k=n+1}^{2n}{2n choose k}p^{k}q^{2n-k+2}=sum_{k=n+3}^{2n+2}{2n choose k-2}p^{k}q^{2n+2-k}+sum_{k=n+2}^{2n+1}2{2n choose k-1}p^{k}q^{2n+2-k}+sum_{k=n+1}^{2n}{2n choose k}p^{k}q^{2n-k+2}.
              $$



              Therefore, $P_{2n}$ equals
              $$
              sum_{k=n+2}^{2n+2}left({2n choose k}+2{2n choose k-1}+{2n choose k-2}right)p^kq^{2n+2-k}-left({2n choose n}p^{n+2}q^{n}-{2n choose n+1}p^{n+1}q^{n+1}right).
              $$



              We also know that
              $$
              {2n choose k}+2{2n choose k-1}+{2n choose k-2}=left({2n choose k}+{2n choose k-1}right)+left({2n choose k-1}+{2n choose k-2}right)={2n+1 choose k}+{2n+1 choose k-1}={2n+2 choose k},
              $$

              so we obtain
              $$
              P_{2n}=sum_{k=n+2}^{2n+2}{2n+2 choose k}p^kq^{2n+2-k}=P_{2n+2}-left({2n choose n}p^{n+2}q^{n}-{2n choose n+1}p^{n+1}q^{n+1}right).
              $$

              Hence,
              $$
              P_{2n+2}=P_{2n}+{2n choose n}p^{n+2}q^{n}-{2n choose n+1}p^{n+1}q^{n+1},
              $$

              as desired.






              share|cite|improve this answer









              $endgroup$



              We consider $(p+q)^2P_{2n}$ instead of $P_{2n}$ (because then our identity will be homogeneous). Then, we will simply compare coefficents of monomials $p^kq^{2n+2-k}$ in both sides of identity.



              Note that (from equality $p+q=1$)
              $$
              P_{2n}=sum_{k=n+1}^{2n}{2n choose k}p^kq^{2n-k}=sum_{k=n+1}^{2n}{2n choose k}p^kq^{2n-k} (p+q)^2,
              $$

              which equals
              $$
              sum_{k=n+1}^{2n}{2n choose k}p^kq^{2n-k} (p^2+2pq+q^2)=sum_{k=n+1}^{2n}{2n choose k}p^{k+2}q^{2n-k}+sum_{k=n+1}^{2n}2{2n choose k}p^{k+1}q^{2n-k+1}+sum_{k=n+1}^{2n}{2n choose k}p^{k}q^{2n-k+2}=sum_{k=n+3}^{2n+2}{2n choose k-2}p^{k}q^{2n+2-k}+sum_{k=n+2}^{2n+1}2{2n choose k-1}p^{k}q^{2n+2-k}+sum_{k=n+1}^{2n}{2n choose k}p^{k}q^{2n-k+2}.
              $$



              Therefore, $P_{2n}$ equals
              $$
              sum_{k=n+2}^{2n+2}left({2n choose k}+2{2n choose k-1}+{2n choose k-2}right)p^kq^{2n+2-k}-left({2n choose n}p^{n+2}q^{n}-{2n choose n+1}p^{n+1}q^{n+1}right).
              $$



              We also know that
              $$
              {2n choose k}+2{2n choose k-1}+{2n choose k-2}=left({2n choose k}+{2n choose k-1}right)+left({2n choose k-1}+{2n choose k-2}right)={2n+1 choose k}+{2n+1 choose k-1}={2n+2 choose k},
              $$

              so we obtain
              $$
              P_{2n}=sum_{k=n+2}^{2n+2}{2n+2 choose k}p^kq^{2n+2-k}=P_{2n+2}-left({2n choose n}p^{n+2}q^{n}-{2n choose n+1}p^{n+1}q^{n+1}right).
              $$

              Hence,
              $$
              P_{2n+2}=P_{2n}+{2n choose n}p^{n+2}q^{n}-{2n choose n+1}p^{n+1}q^{n+1},
              $$

              as desired.







              share|cite|improve this answer












              share|cite|improve this answer



              share|cite|improve this answer










              answered Jan 13 at 13:10









              richrowrichrow

              1615




              1615























                  1












                  $begingroup$

                  The equality reduces to
                  $$
                  binom{2 n}{n+1} p^{n+1} (1-p)^{n-1} , _2F_1left(1,1-n;n+2;frac{p}{p-1}right)+binom{2 n}{n} p^{n+2} (1-p)^n-binom{2 n}{n+1} p^{n+1} (1-p)^{n+1}=binom{2 (n+1)}{n+2} (1-p)^{-n+2 (n+1)-2} p^{n+2} , _2F_1left(1,-n;n+3;frac{p}{p-1}right)
                  $$

                  Let $w=p/(1-p)$, then the above is
                  $$
                  binom{2 n}{n+1} left((w+1)^2 , _2F_1(1,1-n;n+2;-w)-1right)+w binom{2 n}{n}=w binom{2 (n+1)}{n+2} , _2F_1(1,-n;n+3;-w)
                  $$

                  We can then extract the coefficient for $w^m$ from both side for $m in {0,dots,n}$ to see that this equality holds.






                  share|cite|improve this answer









                  $endgroup$


















                    1












                    $begingroup$

                    The equality reduces to
                    $$
                    binom{2 n}{n+1} p^{n+1} (1-p)^{n-1} , _2F_1left(1,1-n;n+2;frac{p}{p-1}right)+binom{2 n}{n} p^{n+2} (1-p)^n-binom{2 n}{n+1} p^{n+1} (1-p)^{n+1}=binom{2 (n+1)}{n+2} (1-p)^{-n+2 (n+1)-2} p^{n+2} , _2F_1left(1,-n;n+3;frac{p}{p-1}right)
                    $$

                    Let $w=p/(1-p)$, then the above is
                    $$
                    binom{2 n}{n+1} left((w+1)^2 , _2F_1(1,1-n;n+2;-w)-1right)+w binom{2 n}{n}=w binom{2 (n+1)}{n+2} , _2F_1(1,-n;n+3;-w)
                    $$

                    We can then extract the coefficient for $w^m$ from both side for $m in {0,dots,n}$ to see that this equality holds.






                    share|cite|improve this answer









                    $endgroup$
















                      1












                      1








                      1





                      $begingroup$

                      The equality reduces to
                      $$
                      binom{2 n}{n+1} p^{n+1} (1-p)^{n-1} , _2F_1left(1,1-n;n+2;frac{p}{p-1}right)+binom{2 n}{n} p^{n+2} (1-p)^n-binom{2 n}{n+1} p^{n+1} (1-p)^{n+1}=binom{2 (n+1)}{n+2} (1-p)^{-n+2 (n+1)-2} p^{n+2} , _2F_1left(1,-n;n+3;frac{p}{p-1}right)
                      $$

                      Let $w=p/(1-p)$, then the above is
                      $$
                      binom{2 n}{n+1} left((w+1)^2 , _2F_1(1,1-n;n+2;-w)-1right)+w binom{2 n}{n}=w binom{2 (n+1)}{n+2} , _2F_1(1,-n;n+3;-w)
                      $$

                      We can then extract the coefficient for $w^m$ from both side for $m in {0,dots,n}$ to see that this equality holds.






                      share|cite|improve this answer









                      $endgroup$



                      The equality reduces to
                      $$
                      binom{2 n}{n+1} p^{n+1} (1-p)^{n-1} , _2F_1left(1,1-n;n+2;frac{p}{p-1}right)+binom{2 n}{n} p^{n+2} (1-p)^n-binom{2 n}{n+1} p^{n+1} (1-p)^{n+1}=binom{2 (n+1)}{n+2} (1-p)^{-n+2 (n+1)-2} p^{n+2} , _2F_1left(1,-n;n+3;frac{p}{p-1}right)
                      $$

                      Let $w=p/(1-p)$, then the above is
                      $$
                      binom{2 n}{n+1} left((w+1)^2 , _2F_1(1,1-n;n+2;-w)-1right)+w binom{2 n}{n}=w binom{2 (n+1)}{n+2} , _2F_1(1,-n;n+3;-w)
                      $$

                      We can then extract the coefficient for $w^m$ from both side for $m in {0,dots,n}$ to see that this equality holds.







                      share|cite|improve this answer












                      share|cite|improve this answer



                      share|cite|improve this answer










                      answered Jan 13 at 11:23









                      ablmfablmf

                      2,52142352




                      2,52142352























                          1












                          $begingroup$

                          Forget the requirement that $0<p<q<1$; it is unnecessary. So let $p$ and $q$
                          be any reals satisfying $p+q=1$.



                          Also, let me extend the stage a little bit:




                          Definition. Let $m$ and $n$ be integers such that $ngeq mgeq0$. Then, we
                          let
                          begin{equation}
                          Q_{n,m}=sum_{k=m}^{n}dbinom{n}{k}p^{k}q^{n-k}.
                          end{equation}




                          With this definition, your $P_{2n}$ is $Q_{2n,n+1}$, while your $P_{2n+2}$ is
                          $Q_{2n+2,n+2}$. Thus, your claim becomes:




                          Theorem 1. Let $n$ be a nonnegative integer. Then,
                          begin{equation}
                          Q_{2n+2,n+2}=Q_{2n,n+1}+dbinom{2n}{n}p^{n+2}q^{n}-dbinom{2n}{n+1}
                          p^{n+1}q^{n+1}.
                          end{equation}




                          I shall derive this from the following:




                          Lemma 2. Let $m$ and $n$ be integers such that $ngeq mgeq1$. Then,
                          begin{equation}
                          Q_{n,m}=Q_{n-1,m-1}-dbinom{n-1}{m-1}p^{m-1}q^{n-m+1}.
                          end{equation}




                          Proof of Lemma 2. From $ngeq mgeq1$, we obtain $n-1geq m-1geq0$. Now,
                          the definition of $Q_{n-1,m-1}$ yields
                          begin{equation}
                          Q_{n-1,m-1}=sum_{k=m-1}^{n-1}dbinom{n-1}{k}p^{k}underbrace{q^{n-1-k}
                          }_{=q^{n-k-1}}=sum_{k=m-1}^{n-1}dbinom{n-1}{k}p^{k}q^{n-k-1}.
                          label{darij1.pf.l2.0}
                          tag{1}
                          end{equation}



                          But the definition of $Q_{n,m}$ yields
                          begin{align}
                          Q_{n,m} & =sum_{k=m}^{n}underbrace{dbinom{n}{k}}_{substack{=dbinom
                          {n-1}{k-1}+dbinom{n-1}{k}\text{(by the recurrence of the binomial
                          coefficients)}}}p^{k}q^{n-k}=sum_{k=m}^{n}left( dbinom{n-1}{k-1}
                          +dbinom{n-1}{k}right) p^{k}q^{n-k}nonumber\
                          & =sum_{k=m}^{n}dbinom{n-1}{k-1}p^{k}q^{n-k}+sum_{k=m}^{n}dbinom{n-1}
                          {k}p^{k}q^{n-k}nonumber\
                          & =sum_{k=m-1}^{n-1}underbrace{dbinom{n-1}{left( k+1right) -1}
                          }_{=dbinom{n-1}{k}}p^{k+1}underbrace{q^{n-left( k+1right) }}
                          _{=q^{n-k-1}}+sum_{k=m}^{n}dbinom{n-1}{k}p^{k}q^{n-k}nonumber\
                          & qquadleft( text{here, we have substituted }k+1text{ for }ktext{ in
                          the first sum}right) nonumber\
                          & =sum_{k=m-1}^{n-1}dbinom{n-1}{k}p^{k+1}q^{n-k-1}+sum_{k=m}^{n}
                          dbinom{n-1}{k}p^{k}q^{n-k}.
                          label{darij1.pf.l2.1}
                          tag{2}
                          end{align}



                          But we have
                          begin{equation}
                          sum_{k=m-1}^{n}dbinom{n-1}{k}p^{k}q^{n-k}=sum_{k=m}^{n}dbinom{n-1}{k}
                          p^{k}q^{n-k}+dbinom{n-1}{m-1}p^{m-1}q^{n-left( m-1right) }
                          end{equation}

                          and
                          begin{align*}
                          sum_{k=m-1}^{n}dbinom{n-1}{k}p^{k}q^{n-k} & =sum_{k=m-1}^{n-1}dbinom
                          {n-1}{k}p^{k}q^{n-k}+underbrace{dbinom{n-1}{n}}_{substack{=0\text{(since
                          }n-1geq0text{ and }n-1<ntext{)}}}p^{n}q^{n-n}\
                          & =sum_{k=m-1}^{n-1}dbinom{n-1}{k}p^{k}q^{n-k}.
                          end{align*}

                          Comparing these two equalities, we obtain
                          begin{equation}
                          sum_{k=m}^{n}dbinom{n-1}{k}p^{k}q^{n-k}+dbinom{n-1}{m-1}p^{m-1}q^{n-left(
                          m-1right) }=sum_{k=m-1}^{n-1}dbinom{n-1}{k}p^{k}q^{n-k}.
                          end{equation}

                          Thus,
                          begin{equation}
                          sum_{k=m}^{n}dbinom{n-1}{k}p^{k}q^{n-k}=sum_{k=m-1}^{n-1}dbinom{n-1}
                          {k}p^{k}q^{n-k}-dbinom{n-1}{m-1}p^{m-1}q^{n-left( m-1right) }.
                          end{equation}

                          Hence, eqref{darij1.pf.l2.1} becomes
                          begin{align*}
                          Q_{n,m} & =sum_{k=m-1}^{n-1}dbinom{n-1}{k}p^{k+1}q^{n-k-1}+underbrace{sum
                          _{k=m}^{n}dbinom{n-1}{k}p^{k}q^{n-k}}_{=sum_{k=m-1}^{n-1}dbinom{n-1}
                          {k}p^{k}q^{n-k}-dbinom{n-1}{m-1}p^{m-1}q^{n-left( m-1right) }}\
                          & =underbrace{sum_{k=m-1}^{n-1}dbinom{n-1}{k}p^{k+1}q^{n-k-1}+sum
                          _{k=m-1}^{n-1}dbinom{n-1}{k}p^{k}q^{n-k}}_{=sum_{k=m-1}^{n-1}dbinom{n-1}
                          {k}left( p^{k+1}q^{n-k-1}+p^{k}q^{n-k}right) }-dbinom{n-1}{m-1}
                          p^{m-1}underbrace{q^{n-left( m-1right) }}_{=q^{n-m+1}}\
                          & =sum_{k=m-1}^{n-1}dbinom{n-1}{k}left( underbrace{p^{k+1}}_{=pp^{k}
                          }q^{n-k-1}+p^{k}underbrace{q^{n-k}}_{=qq^{n-k-1}}right) -dbinom{n-1}
                          {m-1}p^{m-1}q^{n-m+1}\
                          & =sum_{k=m-1}^{n-1}dbinom{n-1}{k}underbrace{left( pp^{k}q^{n-k-1}
                          +p^{k}qq^{n-k-1}right) }_{substack{=left( p+qright) p^{k}
                          q^{n-k-1}=p^{k}q^{n-k-1}\text{(since }p+q=1text{)}}}-dbinom{n-1}
                          {m-1}p^{m-1}q^{n-m+1}\
                          & =underbrace{sum_{k=m-1}^{n-1}dbinom{n-1}{k}p^{k}q^{n-k-1}}
                          _{substack{=Q_{n-1,m-1}\text{(by eqref{darij1.pf.l2.0})}}}-dbinom
                          {n-1}{m-1}p^{m-1}q^{n-m+1}\
                          & =Q_{n-1,m-1}-dbinom{n-1}{m-1}p^{m-1}q^{n-m+1}.
                          end{align*}

                          This proves Lemma 2. $blacksquare$



                          Proof of Theorem 1. We have $left( 1-q-pqright) -p^{2}=-left(
                          p+1right) underbrace{left( p+q-1right) }_{substack{=0\text{(since
                          }p+q=1text{)}}}=0$
                          , thus $1-q-pq=p^{2}$.



                          Lemma 2 (applied to $2n+2$ and $n+2$ instead of $n$ and $m$) yields
                          begin{align}
                          Q_{2n+2,n+2} & =underbrace{Q_{2n+1,n+1}}_{substack{=Q_{2n,n}-dbinom{2n}
                          {n}p^{n}q^{left( 2n+1right) -left( n+1right) +1}\text{(by Lemma 2,
                          applied to }2n+1text{ and }n+1\text{instead of }ntext{ and }mtext{)}
                          }}-underbrace{dbinom{2n+1}{n+1}}_{substack{=dbinom{2n}{n}+dbinom{2n}
                          {n+1}\text{(by the recurrence of the binomial coefficients)}}}p^{n+1}
                          underbrace{q^{left( 2n+2right) -left( n+2right) +1}}_{=q^{n+1}
                          }nonumber\
                          & =Q_{2n,n}-dbinom{2n}{n}p^{n}underbrace{q^{left( 2n+1right) -left(
                          n+1right) +1}}_{=q^{n+1}}-underbrace{left( dbinom{2n}{n}+dbinom
                          {2n}{n+1}right) p^{n+1}q^{n+1}}_{=dbinom{2n}{n}p^{n+1}q^{n+1}+dbinom
                          {2n}{n+1}p^{n+1}q^{n+1}}nonumber\
                          & =Q_{2n,n}-dbinom{2n}{n}p^{n}q^{n+1}-left( dbinom{2n}{n}p^{n+1}
                          q^{n+1}+dbinom{2n}{n+1}p^{n+1}q^{n+1}right) nonumber\
                          & =Q_{2n,n}-dbinom{2n}{n}p^{n}q^{n+1}-dbinom{2n}{n}p^{n+1}q^{n+1}
                          -dbinom{2n}{n+1}p^{n+1}q^{n+1}.
                          label{darij1.pf.t1.1}
                          tag{3}
                          end{align}

                          But the definition of $Q_{2n,n+1}$ yields $Q_{2n,n+1}=sum_{k=n+1}^{2n}
                          dbinom{2n}{k}p^{k}q^{2n-k}$
                          . Meanwhile, the definition of $Q_{2n,n}$ yields
                          begin{align*}
                          Q_{2n,n} & =sum_{k=n}^{2n}dbinom{2n}{k}p^{k}q^{2n-k}=underbrace{sum
                          _{k=n+1}^{2n}dbinom{2n}{k}p^{k}q^{2n-k}}_{=Q_{2n,n+1}}+dbinom{2n}{n}
                          p^{n}underbrace{q^{2n-n}}_{=q^{n}}\
                          & =Q_{2n,n+1}+dbinom{2n}{n}p^{n}q^{n}.
                          end{align*}

                          Thus,
                          eqref{darij1.pf.t1.1} becomes
                          begin{align*}
                          Q_{2n+2,n+2} & =underbrace{Q_{2n,n}}_{=Q_{2n,n+1}+dbinom{2n}{n}p^{n}q^{n}
                          }-dbinom{2n}{n}p^{n}q^{n+1}-dbinom{2n}{n}p^{n+1}q^{n+1}-dbinom{2n}
                          {n+1}p^{n+1}q^{n+1}\
                          & =Q_{2n,n+1}+dbinom{2n}{n}p^{n}q^{n}-dbinom{2n}{n}p^{n}q^{n+1}-dbinom
                          {2n}{n}p^{n+1}q^{n+1}-dbinom{2n}{n+1}p^{n+1}q^{n+1}\
                          & =Q_{2n,n+1}+dbinom{2n}{n}underbrace{left( p^{n}q^{n}-p^{n}
                          q^{n+1}-p^{n+1}q^{n+1}right) }_{=left( 1-q-pqright) p^{n}q^{n}}
                          -dbinom{2n}{n+1}p^{n+1}q^{n+1}\
                          & =Q_{2n,n+1}+dbinom{2n}{n}underbrace{left( 1-q-pqright) }_{=p^{2}}
                          p^{n}q^{n}-dbinom{2n}{n+1}p^{n+1}q^{n+1}\
                          & =Q_{2n,n+1}+dbinom{2n}{n}underbrace{p^{2}p^{n}}_{=p^{n+2}}q^{n}
                          -dbinom{2n}{n+1}p^{n+1}q^{n+1}\
                          & =Q_{2n,n+1}+dbinom{2n}{n}p^{n+2}q^{n}-dbinom{2n}{n+1}p^{n+1}q^{n+1}.
                          end{align*}

                          This proves Theorem 1. $blacksquare$






                          share|cite|improve this answer









                          $endgroup$


















                            1












                            $begingroup$

                            Forget the requirement that $0<p<q<1$; it is unnecessary. So let $p$ and $q$
                            be any reals satisfying $p+q=1$.



                            Also, let me extend the stage a little bit:




                            Definition. Let $m$ and $n$ be integers such that $ngeq mgeq0$. Then, we
                            let
                            begin{equation}
                            Q_{n,m}=sum_{k=m}^{n}dbinom{n}{k}p^{k}q^{n-k}.
                            end{equation}




                            With this definition, your $P_{2n}$ is $Q_{2n,n+1}$, while your $P_{2n+2}$ is
                            $Q_{2n+2,n+2}$. Thus, your claim becomes:




                            Theorem 1. Let $n$ be a nonnegative integer. Then,
                            begin{equation}
                            Q_{2n+2,n+2}=Q_{2n,n+1}+dbinom{2n}{n}p^{n+2}q^{n}-dbinom{2n}{n+1}
                            p^{n+1}q^{n+1}.
                            end{equation}




                            I shall derive this from the following:




                            Lemma 2. Let $m$ and $n$ be integers such that $ngeq mgeq1$. Then,
                            begin{equation}
                            Q_{n,m}=Q_{n-1,m-1}-dbinom{n-1}{m-1}p^{m-1}q^{n-m+1}.
                            end{equation}




                            Proof of Lemma 2. From $ngeq mgeq1$, we obtain $n-1geq m-1geq0$. Now,
                            the definition of $Q_{n-1,m-1}$ yields
                            begin{equation}
                            Q_{n-1,m-1}=sum_{k=m-1}^{n-1}dbinom{n-1}{k}p^{k}underbrace{q^{n-1-k}
                            }_{=q^{n-k-1}}=sum_{k=m-1}^{n-1}dbinom{n-1}{k}p^{k}q^{n-k-1}.
                            label{darij1.pf.l2.0}
                            tag{1}
                            end{equation}



                            But the definition of $Q_{n,m}$ yields
                            begin{align}
                            Q_{n,m} & =sum_{k=m}^{n}underbrace{dbinom{n}{k}}_{substack{=dbinom
                            {n-1}{k-1}+dbinom{n-1}{k}\text{(by the recurrence of the binomial
                            coefficients)}}}p^{k}q^{n-k}=sum_{k=m}^{n}left( dbinom{n-1}{k-1}
                            +dbinom{n-1}{k}right) p^{k}q^{n-k}nonumber\
                            & =sum_{k=m}^{n}dbinom{n-1}{k-1}p^{k}q^{n-k}+sum_{k=m}^{n}dbinom{n-1}
                            {k}p^{k}q^{n-k}nonumber\
                            & =sum_{k=m-1}^{n-1}underbrace{dbinom{n-1}{left( k+1right) -1}
                            }_{=dbinom{n-1}{k}}p^{k+1}underbrace{q^{n-left( k+1right) }}
                            _{=q^{n-k-1}}+sum_{k=m}^{n}dbinom{n-1}{k}p^{k}q^{n-k}nonumber\
                            & qquadleft( text{here, we have substituted }k+1text{ for }ktext{ in
                            the first sum}right) nonumber\
                            & =sum_{k=m-1}^{n-1}dbinom{n-1}{k}p^{k+1}q^{n-k-1}+sum_{k=m}^{n}
                            dbinom{n-1}{k}p^{k}q^{n-k}.
                            label{darij1.pf.l2.1}
                            tag{2}
                            end{align}



                            But we have
                            begin{equation}
                            sum_{k=m-1}^{n}dbinom{n-1}{k}p^{k}q^{n-k}=sum_{k=m}^{n}dbinom{n-1}{k}
                            p^{k}q^{n-k}+dbinom{n-1}{m-1}p^{m-1}q^{n-left( m-1right) }
                            end{equation}

                            and
                            begin{align*}
                            sum_{k=m-1}^{n}dbinom{n-1}{k}p^{k}q^{n-k} & =sum_{k=m-1}^{n-1}dbinom
                            {n-1}{k}p^{k}q^{n-k}+underbrace{dbinom{n-1}{n}}_{substack{=0\text{(since
                            }n-1geq0text{ and }n-1<ntext{)}}}p^{n}q^{n-n}\
                            & =sum_{k=m-1}^{n-1}dbinom{n-1}{k}p^{k}q^{n-k}.
                            end{align*}

                            Comparing these two equalities, we obtain
                            begin{equation}
                            sum_{k=m}^{n}dbinom{n-1}{k}p^{k}q^{n-k}+dbinom{n-1}{m-1}p^{m-1}q^{n-left(
                            m-1right) }=sum_{k=m-1}^{n-1}dbinom{n-1}{k}p^{k}q^{n-k}.
                            end{equation}

                            Thus,
                            begin{equation}
                            sum_{k=m}^{n}dbinom{n-1}{k}p^{k}q^{n-k}=sum_{k=m-1}^{n-1}dbinom{n-1}
                            {k}p^{k}q^{n-k}-dbinom{n-1}{m-1}p^{m-1}q^{n-left( m-1right) }.
                            end{equation}

                            Hence, eqref{darij1.pf.l2.1} becomes
                            begin{align*}
                            Q_{n,m} & =sum_{k=m-1}^{n-1}dbinom{n-1}{k}p^{k+1}q^{n-k-1}+underbrace{sum
                            _{k=m}^{n}dbinom{n-1}{k}p^{k}q^{n-k}}_{=sum_{k=m-1}^{n-1}dbinom{n-1}
                            {k}p^{k}q^{n-k}-dbinom{n-1}{m-1}p^{m-1}q^{n-left( m-1right) }}\
                            & =underbrace{sum_{k=m-1}^{n-1}dbinom{n-1}{k}p^{k+1}q^{n-k-1}+sum
                            _{k=m-1}^{n-1}dbinom{n-1}{k}p^{k}q^{n-k}}_{=sum_{k=m-1}^{n-1}dbinom{n-1}
                            {k}left( p^{k+1}q^{n-k-1}+p^{k}q^{n-k}right) }-dbinom{n-1}{m-1}
                            p^{m-1}underbrace{q^{n-left( m-1right) }}_{=q^{n-m+1}}\
                            & =sum_{k=m-1}^{n-1}dbinom{n-1}{k}left( underbrace{p^{k+1}}_{=pp^{k}
                            }q^{n-k-1}+p^{k}underbrace{q^{n-k}}_{=qq^{n-k-1}}right) -dbinom{n-1}
                            {m-1}p^{m-1}q^{n-m+1}\
                            & =sum_{k=m-1}^{n-1}dbinom{n-1}{k}underbrace{left( pp^{k}q^{n-k-1}
                            +p^{k}qq^{n-k-1}right) }_{substack{=left( p+qright) p^{k}
                            q^{n-k-1}=p^{k}q^{n-k-1}\text{(since }p+q=1text{)}}}-dbinom{n-1}
                            {m-1}p^{m-1}q^{n-m+1}\
                            & =underbrace{sum_{k=m-1}^{n-1}dbinom{n-1}{k}p^{k}q^{n-k-1}}
                            _{substack{=Q_{n-1,m-1}\text{(by eqref{darij1.pf.l2.0})}}}-dbinom
                            {n-1}{m-1}p^{m-1}q^{n-m+1}\
                            & =Q_{n-1,m-1}-dbinom{n-1}{m-1}p^{m-1}q^{n-m+1}.
                            end{align*}

                            This proves Lemma 2. $blacksquare$



                            Proof of Theorem 1. We have $left( 1-q-pqright) -p^{2}=-left(
                            p+1right) underbrace{left( p+q-1right) }_{substack{=0\text{(since
                            }p+q=1text{)}}}=0$
                            , thus $1-q-pq=p^{2}$.



                            Lemma 2 (applied to $2n+2$ and $n+2$ instead of $n$ and $m$) yields
                            begin{align}
                            Q_{2n+2,n+2} & =underbrace{Q_{2n+1,n+1}}_{substack{=Q_{2n,n}-dbinom{2n}
                            {n}p^{n}q^{left( 2n+1right) -left( n+1right) +1}\text{(by Lemma 2,
                            applied to }2n+1text{ and }n+1\text{instead of }ntext{ and }mtext{)}
                            }}-underbrace{dbinom{2n+1}{n+1}}_{substack{=dbinom{2n}{n}+dbinom{2n}
                            {n+1}\text{(by the recurrence of the binomial coefficients)}}}p^{n+1}
                            underbrace{q^{left( 2n+2right) -left( n+2right) +1}}_{=q^{n+1}
                            }nonumber\
                            & =Q_{2n,n}-dbinom{2n}{n}p^{n}underbrace{q^{left( 2n+1right) -left(
                            n+1right) +1}}_{=q^{n+1}}-underbrace{left( dbinom{2n}{n}+dbinom
                            {2n}{n+1}right) p^{n+1}q^{n+1}}_{=dbinom{2n}{n}p^{n+1}q^{n+1}+dbinom
                            {2n}{n+1}p^{n+1}q^{n+1}}nonumber\
                            & =Q_{2n,n}-dbinom{2n}{n}p^{n}q^{n+1}-left( dbinom{2n}{n}p^{n+1}
                            q^{n+1}+dbinom{2n}{n+1}p^{n+1}q^{n+1}right) nonumber\
                            & =Q_{2n,n}-dbinom{2n}{n}p^{n}q^{n+1}-dbinom{2n}{n}p^{n+1}q^{n+1}
                            -dbinom{2n}{n+1}p^{n+1}q^{n+1}.
                            label{darij1.pf.t1.1}
                            tag{3}
                            end{align}

                            But the definition of $Q_{2n,n+1}$ yields $Q_{2n,n+1}=sum_{k=n+1}^{2n}
                            dbinom{2n}{k}p^{k}q^{2n-k}$
                            . Meanwhile, the definition of $Q_{2n,n}$ yields
                            begin{align*}
                            Q_{2n,n} & =sum_{k=n}^{2n}dbinom{2n}{k}p^{k}q^{2n-k}=underbrace{sum
                            _{k=n+1}^{2n}dbinom{2n}{k}p^{k}q^{2n-k}}_{=Q_{2n,n+1}}+dbinom{2n}{n}
                            p^{n}underbrace{q^{2n-n}}_{=q^{n}}\
                            & =Q_{2n,n+1}+dbinom{2n}{n}p^{n}q^{n}.
                            end{align*}

                            Thus,
                            eqref{darij1.pf.t1.1} becomes
                            begin{align*}
                            Q_{2n+2,n+2} & =underbrace{Q_{2n,n}}_{=Q_{2n,n+1}+dbinom{2n}{n}p^{n}q^{n}
                            }-dbinom{2n}{n}p^{n}q^{n+1}-dbinom{2n}{n}p^{n+1}q^{n+1}-dbinom{2n}
                            {n+1}p^{n+1}q^{n+1}\
                            & =Q_{2n,n+1}+dbinom{2n}{n}p^{n}q^{n}-dbinom{2n}{n}p^{n}q^{n+1}-dbinom
                            {2n}{n}p^{n+1}q^{n+1}-dbinom{2n}{n+1}p^{n+1}q^{n+1}\
                            & =Q_{2n,n+1}+dbinom{2n}{n}underbrace{left( p^{n}q^{n}-p^{n}
                            q^{n+1}-p^{n+1}q^{n+1}right) }_{=left( 1-q-pqright) p^{n}q^{n}}
                            -dbinom{2n}{n+1}p^{n+1}q^{n+1}\
                            & =Q_{2n,n+1}+dbinom{2n}{n}underbrace{left( 1-q-pqright) }_{=p^{2}}
                            p^{n}q^{n}-dbinom{2n}{n+1}p^{n+1}q^{n+1}\
                            & =Q_{2n,n+1}+dbinom{2n}{n}underbrace{p^{2}p^{n}}_{=p^{n+2}}q^{n}
                            -dbinom{2n}{n+1}p^{n+1}q^{n+1}\
                            & =Q_{2n,n+1}+dbinom{2n}{n}p^{n+2}q^{n}-dbinom{2n}{n+1}p^{n+1}q^{n+1}.
                            end{align*}

                            This proves Theorem 1. $blacksquare$






                            share|cite|improve this answer









                            $endgroup$
















                              1












                              1








                              1





                              $begingroup$

                              Forget the requirement that $0<p<q<1$; it is unnecessary. So let $p$ and $q$
                              be any reals satisfying $p+q=1$.



                              Also, let me extend the stage a little bit:




                              Definition. Let $m$ and $n$ be integers such that $ngeq mgeq0$. Then, we
                              let
                              begin{equation}
                              Q_{n,m}=sum_{k=m}^{n}dbinom{n}{k}p^{k}q^{n-k}.
                              end{equation}




                              With this definition, your $P_{2n}$ is $Q_{2n,n+1}$, while your $P_{2n+2}$ is
                              $Q_{2n+2,n+2}$. Thus, your claim becomes:




                              Theorem 1. Let $n$ be a nonnegative integer. Then,
                              begin{equation}
                              Q_{2n+2,n+2}=Q_{2n,n+1}+dbinom{2n}{n}p^{n+2}q^{n}-dbinom{2n}{n+1}
                              p^{n+1}q^{n+1}.
                              end{equation}




                              I shall derive this from the following:




                              Lemma 2. Let $m$ and $n$ be integers such that $ngeq mgeq1$. Then,
                              begin{equation}
                              Q_{n,m}=Q_{n-1,m-1}-dbinom{n-1}{m-1}p^{m-1}q^{n-m+1}.
                              end{equation}




                              Proof of Lemma 2. From $ngeq mgeq1$, we obtain $n-1geq m-1geq0$. Now,
                              the definition of $Q_{n-1,m-1}$ yields
                              begin{equation}
                              Q_{n-1,m-1}=sum_{k=m-1}^{n-1}dbinom{n-1}{k}p^{k}underbrace{q^{n-1-k}
                              }_{=q^{n-k-1}}=sum_{k=m-1}^{n-1}dbinom{n-1}{k}p^{k}q^{n-k-1}.
                              label{darij1.pf.l2.0}
                              tag{1}
                              end{equation}



                              But the definition of $Q_{n,m}$ yields
                              begin{align}
                              Q_{n,m} & =sum_{k=m}^{n}underbrace{dbinom{n}{k}}_{substack{=dbinom
                              {n-1}{k-1}+dbinom{n-1}{k}\text{(by the recurrence of the binomial
                              coefficients)}}}p^{k}q^{n-k}=sum_{k=m}^{n}left( dbinom{n-1}{k-1}
                              +dbinom{n-1}{k}right) p^{k}q^{n-k}nonumber\
                              & =sum_{k=m}^{n}dbinom{n-1}{k-1}p^{k}q^{n-k}+sum_{k=m}^{n}dbinom{n-1}
                              {k}p^{k}q^{n-k}nonumber\
                              & =sum_{k=m-1}^{n-1}underbrace{dbinom{n-1}{left( k+1right) -1}
                              }_{=dbinom{n-1}{k}}p^{k+1}underbrace{q^{n-left( k+1right) }}
                              _{=q^{n-k-1}}+sum_{k=m}^{n}dbinom{n-1}{k}p^{k}q^{n-k}nonumber\
                              & qquadleft( text{here, we have substituted }k+1text{ for }ktext{ in
                              the first sum}right) nonumber\
                              & =sum_{k=m-1}^{n-1}dbinom{n-1}{k}p^{k+1}q^{n-k-1}+sum_{k=m}^{n}
                              dbinom{n-1}{k}p^{k}q^{n-k}.
                              label{darij1.pf.l2.1}
                              tag{2}
                              end{align}



                              But we have
                              begin{equation}
                              sum_{k=m-1}^{n}dbinom{n-1}{k}p^{k}q^{n-k}=sum_{k=m}^{n}dbinom{n-1}{k}
                              p^{k}q^{n-k}+dbinom{n-1}{m-1}p^{m-1}q^{n-left( m-1right) }
                              end{equation}

                              and
                              begin{align*}
                              sum_{k=m-1}^{n}dbinom{n-1}{k}p^{k}q^{n-k} & =sum_{k=m-1}^{n-1}dbinom
                              {n-1}{k}p^{k}q^{n-k}+underbrace{dbinom{n-1}{n}}_{substack{=0\text{(since
                              }n-1geq0text{ and }n-1<ntext{)}}}p^{n}q^{n-n}\
                              & =sum_{k=m-1}^{n-1}dbinom{n-1}{k}p^{k}q^{n-k}.
                              end{align*}

                              Comparing these two equalities, we obtain
                              begin{equation}
                              sum_{k=m}^{n}dbinom{n-1}{k}p^{k}q^{n-k}+dbinom{n-1}{m-1}p^{m-1}q^{n-left(
                              m-1right) }=sum_{k=m-1}^{n-1}dbinom{n-1}{k}p^{k}q^{n-k}.
                              end{equation}

                              Thus,
                              begin{equation}
                              sum_{k=m}^{n}dbinom{n-1}{k}p^{k}q^{n-k}=sum_{k=m-1}^{n-1}dbinom{n-1}
                              {k}p^{k}q^{n-k}-dbinom{n-1}{m-1}p^{m-1}q^{n-left( m-1right) }.
                              end{equation}

                              Hence, eqref{darij1.pf.l2.1} becomes
                              begin{align*}
                              Q_{n,m} & =sum_{k=m-1}^{n-1}dbinom{n-1}{k}p^{k+1}q^{n-k-1}+underbrace{sum
                              _{k=m}^{n}dbinom{n-1}{k}p^{k}q^{n-k}}_{=sum_{k=m-1}^{n-1}dbinom{n-1}
                              {k}p^{k}q^{n-k}-dbinom{n-1}{m-1}p^{m-1}q^{n-left( m-1right) }}\
                              & =underbrace{sum_{k=m-1}^{n-1}dbinom{n-1}{k}p^{k+1}q^{n-k-1}+sum
                              _{k=m-1}^{n-1}dbinom{n-1}{k}p^{k}q^{n-k}}_{=sum_{k=m-1}^{n-1}dbinom{n-1}
                              {k}left( p^{k+1}q^{n-k-1}+p^{k}q^{n-k}right) }-dbinom{n-1}{m-1}
                              p^{m-1}underbrace{q^{n-left( m-1right) }}_{=q^{n-m+1}}\
                              & =sum_{k=m-1}^{n-1}dbinom{n-1}{k}left( underbrace{p^{k+1}}_{=pp^{k}
                              }q^{n-k-1}+p^{k}underbrace{q^{n-k}}_{=qq^{n-k-1}}right) -dbinom{n-1}
                              {m-1}p^{m-1}q^{n-m+1}\
                              & =sum_{k=m-1}^{n-1}dbinom{n-1}{k}underbrace{left( pp^{k}q^{n-k-1}
                              +p^{k}qq^{n-k-1}right) }_{substack{=left( p+qright) p^{k}
                              q^{n-k-1}=p^{k}q^{n-k-1}\text{(since }p+q=1text{)}}}-dbinom{n-1}
                              {m-1}p^{m-1}q^{n-m+1}\
                              & =underbrace{sum_{k=m-1}^{n-1}dbinom{n-1}{k}p^{k}q^{n-k-1}}
                              _{substack{=Q_{n-1,m-1}\text{(by eqref{darij1.pf.l2.0})}}}-dbinom
                              {n-1}{m-1}p^{m-1}q^{n-m+1}\
                              & =Q_{n-1,m-1}-dbinom{n-1}{m-1}p^{m-1}q^{n-m+1}.
                              end{align*}

                              This proves Lemma 2. $blacksquare$



                              Proof of Theorem 1. We have $left( 1-q-pqright) -p^{2}=-left(
                              p+1right) underbrace{left( p+q-1right) }_{substack{=0\text{(since
                              }p+q=1text{)}}}=0$
                              , thus $1-q-pq=p^{2}$.



                              Lemma 2 (applied to $2n+2$ and $n+2$ instead of $n$ and $m$) yields
                              begin{align}
                              Q_{2n+2,n+2} & =underbrace{Q_{2n+1,n+1}}_{substack{=Q_{2n,n}-dbinom{2n}
                              {n}p^{n}q^{left( 2n+1right) -left( n+1right) +1}\text{(by Lemma 2,
                              applied to }2n+1text{ and }n+1\text{instead of }ntext{ and }mtext{)}
                              }}-underbrace{dbinom{2n+1}{n+1}}_{substack{=dbinom{2n}{n}+dbinom{2n}
                              {n+1}\text{(by the recurrence of the binomial coefficients)}}}p^{n+1}
                              underbrace{q^{left( 2n+2right) -left( n+2right) +1}}_{=q^{n+1}
                              }nonumber\
                              & =Q_{2n,n}-dbinom{2n}{n}p^{n}underbrace{q^{left( 2n+1right) -left(
                              n+1right) +1}}_{=q^{n+1}}-underbrace{left( dbinom{2n}{n}+dbinom
                              {2n}{n+1}right) p^{n+1}q^{n+1}}_{=dbinom{2n}{n}p^{n+1}q^{n+1}+dbinom
                              {2n}{n+1}p^{n+1}q^{n+1}}nonumber\
                              & =Q_{2n,n}-dbinom{2n}{n}p^{n}q^{n+1}-left( dbinom{2n}{n}p^{n+1}
                              q^{n+1}+dbinom{2n}{n+1}p^{n+1}q^{n+1}right) nonumber\
                              & =Q_{2n,n}-dbinom{2n}{n}p^{n}q^{n+1}-dbinom{2n}{n}p^{n+1}q^{n+1}
                              -dbinom{2n}{n+1}p^{n+1}q^{n+1}.
                              label{darij1.pf.t1.1}
                              tag{3}
                              end{align}

                              But the definition of $Q_{2n,n+1}$ yields $Q_{2n,n+1}=sum_{k=n+1}^{2n}
                              dbinom{2n}{k}p^{k}q^{2n-k}$
                              . Meanwhile, the definition of $Q_{2n,n}$ yields
                              begin{align*}
                              Q_{2n,n} & =sum_{k=n}^{2n}dbinom{2n}{k}p^{k}q^{2n-k}=underbrace{sum
                              _{k=n+1}^{2n}dbinom{2n}{k}p^{k}q^{2n-k}}_{=Q_{2n,n+1}}+dbinom{2n}{n}
                              p^{n}underbrace{q^{2n-n}}_{=q^{n}}\
                              & =Q_{2n,n+1}+dbinom{2n}{n}p^{n}q^{n}.
                              end{align*}

                              Thus,
                              eqref{darij1.pf.t1.1} becomes
                              begin{align*}
                              Q_{2n+2,n+2} & =underbrace{Q_{2n,n}}_{=Q_{2n,n+1}+dbinom{2n}{n}p^{n}q^{n}
                              }-dbinom{2n}{n}p^{n}q^{n+1}-dbinom{2n}{n}p^{n+1}q^{n+1}-dbinom{2n}
                              {n+1}p^{n+1}q^{n+1}\
                              & =Q_{2n,n+1}+dbinom{2n}{n}p^{n}q^{n}-dbinom{2n}{n}p^{n}q^{n+1}-dbinom
                              {2n}{n}p^{n+1}q^{n+1}-dbinom{2n}{n+1}p^{n+1}q^{n+1}\
                              & =Q_{2n,n+1}+dbinom{2n}{n}underbrace{left( p^{n}q^{n}-p^{n}
                              q^{n+1}-p^{n+1}q^{n+1}right) }_{=left( 1-q-pqright) p^{n}q^{n}}
                              -dbinom{2n}{n+1}p^{n+1}q^{n+1}\
                              & =Q_{2n,n+1}+dbinom{2n}{n}underbrace{left( 1-q-pqright) }_{=p^{2}}
                              p^{n}q^{n}-dbinom{2n}{n+1}p^{n+1}q^{n+1}\
                              & =Q_{2n,n+1}+dbinom{2n}{n}underbrace{p^{2}p^{n}}_{=p^{n+2}}q^{n}
                              -dbinom{2n}{n+1}p^{n+1}q^{n+1}\
                              & =Q_{2n,n+1}+dbinom{2n}{n}p^{n+2}q^{n}-dbinom{2n}{n+1}p^{n+1}q^{n+1}.
                              end{align*}

                              This proves Theorem 1. $blacksquare$






                              share|cite|improve this answer









                              $endgroup$



                              Forget the requirement that $0<p<q<1$; it is unnecessary. So let $p$ and $q$
                              be any reals satisfying $p+q=1$.



                              Also, let me extend the stage a little bit:




                              Definition. Let $m$ and $n$ be integers such that $ngeq mgeq0$. Then, we
                              let
                              begin{equation}
                              Q_{n,m}=sum_{k=m}^{n}dbinom{n}{k}p^{k}q^{n-k}.
                              end{equation}




                              With this definition, your $P_{2n}$ is $Q_{2n,n+1}$, while your $P_{2n+2}$ is
                              $Q_{2n+2,n+2}$. Thus, your claim becomes:




                              Theorem 1. Let $n$ be a nonnegative integer. Then,
                              begin{equation}
                              Q_{2n+2,n+2}=Q_{2n,n+1}+dbinom{2n}{n}p^{n+2}q^{n}-dbinom{2n}{n+1}
                              p^{n+1}q^{n+1}.
                              end{equation}




                              I shall derive this from the following:




                              Lemma 2. Let $m$ and $n$ be integers such that $ngeq mgeq1$. Then,
                              begin{equation}
                              Q_{n,m}=Q_{n-1,m-1}-dbinom{n-1}{m-1}p^{m-1}q^{n-m+1}.
                              end{equation}




                              Proof of Lemma 2. From $ngeq mgeq1$, we obtain $n-1geq m-1geq0$. Now,
                              the definition of $Q_{n-1,m-1}$ yields
                              begin{equation}
                              Q_{n-1,m-1}=sum_{k=m-1}^{n-1}dbinom{n-1}{k}p^{k}underbrace{q^{n-1-k}
                              }_{=q^{n-k-1}}=sum_{k=m-1}^{n-1}dbinom{n-1}{k}p^{k}q^{n-k-1}.
                              label{darij1.pf.l2.0}
                              tag{1}
                              end{equation}



                              But the definition of $Q_{n,m}$ yields
                              begin{align}
                              Q_{n,m} & =sum_{k=m}^{n}underbrace{dbinom{n}{k}}_{substack{=dbinom
                              {n-1}{k-1}+dbinom{n-1}{k}\text{(by the recurrence of the binomial
                              coefficients)}}}p^{k}q^{n-k}=sum_{k=m}^{n}left( dbinom{n-1}{k-1}
                              +dbinom{n-1}{k}right) p^{k}q^{n-k}nonumber\
                              & =sum_{k=m}^{n}dbinom{n-1}{k-1}p^{k}q^{n-k}+sum_{k=m}^{n}dbinom{n-1}
                              {k}p^{k}q^{n-k}nonumber\
                              & =sum_{k=m-1}^{n-1}underbrace{dbinom{n-1}{left( k+1right) -1}
                              }_{=dbinom{n-1}{k}}p^{k+1}underbrace{q^{n-left( k+1right) }}
                              _{=q^{n-k-1}}+sum_{k=m}^{n}dbinom{n-1}{k}p^{k}q^{n-k}nonumber\
                              & qquadleft( text{here, we have substituted }k+1text{ for }ktext{ in
                              the first sum}right) nonumber\
                              & =sum_{k=m-1}^{n-1}dbinom{n-1}{k}p^{k+1}q^{n-k-1}+sum_{k=m}^{n}
                              dbinom{n-1}{k}p^{k}q^{n-k}.
                              label{darij1.pf.l2.1}
                              tag{2}
                              end{align}



                              But we have
                              begin{equation}
                              sum_{k=m-1}^{n}dbinom{n-1}{k}p^{k}q^{n-k}=sum_{k=m}^{n}dbinom{n-1}{k}
                              p^{k}q^{n-k}+dbinom{n-1}{m-1}p^{m-1}q^{n-left( m-1right) }
                              end{equation}

                              and
                              begin{align*}
                              sum_{k=m-1}^{n}dbinom{n-1}{k}p^{k}q^{n-k} & =sum_{k=m-1}^{n-1}dbinom
                              {n-1}{k}p^{k}q^{n-k}+underbrace{dbinom{n-1}{n}}_{substack{=0\text{(since
                              }n-1geq0text{ and }n-1<ntext{)}}}p^{n}q^{n-n}\
                              & =sum_{k=m-1}^{n-1}dbinom{n-1}{k}p^{k}q^{n-k}.
                              end{align*}

                              Comparing these two equalities, we obtain
                              begin{equation}
                              sum_{k=m}^{n}dbinom{n-1}{k}p^{k}q^{n-k}+dbinom{n-1}{m-1}p^{m-1}q^{n-left(
                              m-1right) }=sum_{k=m-1}^{n-1}dbinom{n-1}{k}p^{k}q^{n-k}.
                              end{equation}

                              Thus,
                              begin{equation}
                              sum_{k=m}^{n}dbinom{n-1}{k}p^{k}q^{n-k}=sum_{k=m-1}^{n-1}dbinom{n-1}
                              {k}p^{k}q^{n-k}-dbinom{n-1}{m-1}p^{m-1}q^{n-left( m-1right) }.
                              end{equation}

                              Hence, eqref{darij1.pf.l2.1} becomes
                              begin{align*}
                              Q_{n,m} & =sum_{k=m-1}^{n-1}dbinom{n-1}{k}p^{k+1}q^{n-k-1}+underbrace{sum
                              _{k=m}^{n}dbinom{n-1}{k}p^{k}q^{n-k}}_{=sum_{k=m-1}^{n-1}dbinom{n-1}
                              {k}p^{k}q^{n-k}-dbinom{n-1}{m-1}p^{m-1}q^{n-left( m-1right) }}\
                              & =underbrace{sum_{k=m-1}^{n-1}dbinom{n-1}{k}p^{k+1}q^{n-k-1}+sum
                              _{k=m-1}^{n-1}dbinom{n-1}{k}p^{k}q^{n-k}}_{=sum_{k=m-1}^{n-1}dbinom{n-1}
                              {k}left( p^{k+1}q^{n-k-1}+p^{k}q^{n-k}right) }-dbinom{n-1}{m-1}
                              p^{m-1}underbrace{q^{n-left( m-1right) }}_{=q^{n-m+1}}\
                              & =sum_{k=m-1}^{n-1}dbinom{n-1}{k}left( underbrace{p^{k+1}}_{=pp^{k}
                              }q^{n-k-1}+p^{k}underbrace{q^{n-k}}_{=qq^{n-k-1}}right) -dbinom{n-1}
                              {m-1}p^{m-1}q^{n-m+1}\
                              & =sum_{k=m-1}^{n-1}dbinom{n-1}{k}underbrace{left( pp^{k}q^{n-k-1}
                              +p^{k}qq^{n-k-1}right) }_{substack{=left( p+qright) p^{k}
                              q^{n-k-1}=p^{k}q^{n-k-1}\text{(since }p+q=1text{)}}}-dbinom{n-1}
                              {m-1}p^{m-1}q^{n-m+1}\
                              & =underbrace{sum_{k=m-1}^{n-1}dbinom{n-1}{k}p^{k}q^{n-k-1}}
                              _{substack{=Q_{n-1,m-1}\text{(by eqref{darij1.pf.l2.0})}}}-dbinom
                              {n-1}{m-1}p^{m-1}q^{n-m+1}\
                              & =Q_{n-1,m-1}-dbinom{n-1}{m-1}p^{m-1}q^{n-m+1}.
                              end{align*}

                              This proves Lemma 2. $blacksquare$



                              Proof of Theorem 1. We have $left( 1-q-pqright) -p^{2}=-left(
                              p+1right) underbrace{left( p+q-1right) }_{substack{=0\text{(since
                              }p+q=1text{)}}}=0$
                              , thus $1-q-pq=p^{2}$.



                              Lemma 2 (applied to $2n+2$ and $n+2$ instead of $n$ and $m$) yields
                              begin{align}
                              Q_{2n+2,n+2} & =underbrace{Q_{2n+1,n+1}}_{substack{=Q_{2n,n}-dbinom{2n}
                              {n}p^{n}q^{left( 2n+1right) -left( n+1right) +1}\text{(by Lemma 2,
                              applied to }2n+1text{ and }n+1\text{instead of }ntext{ and }mtext{)}
                              }}-underbrace{dbinom{2n+1}{n+1}}_{substack{=dbinom{2n}{n}+dbinom{2n}
                              {n+1}\text{(by the recurrence of the binomial coefficients)}}}p^{n+1}
                              underbrace{q^{left( 2n+2right) -left( n+2right) +1}}_{=q^{n+1}
                              }nonumber\
                              & =Q_{2n,n}-dbinom{2n}{n}p^{n}underbrace{q^{left( 2n+1right) -left(
                              n+1right) +1}}_{=q^{n+1}}-underbrace{left( dbinom{2n}{n}+dbinom
                              {2n}{n+1}right) p^{n+1}q^{n+1}}_{=dbinom{2n}{n}p^{n+1}q^{n+1}+dbinom
                              {2n}{n+1}p^{n+1}q^{n+1}}nonumber\
                              & =Q_{2n,n}-dbinom{2n}{n}p^{n}q^{n+1}-left( dbinom{2n}{n}p^{n+1}
                              q^{n+1}+dbinom{2n}{n+1}p^{n+1}q^{n+1}right) nonumber\
                              & =Q_{2n,n}-dbinom{2n}{n}p^{n}q^{n+1}-dbinom{2n}{n}p^{n+1}q^{n+1}
                              -dbinom{2n}{n+1}p^{n+1}q^{n+1}.
                              label{darij1.pf.t1.1}
                              tag{3}
                              end{align}

                              But the definition of $Q_{2n,n+1}$ yields $Q_{2n,n+1}=sum_{k=n+1}^{2n}
                              dbinom{2n}{k}p^{k}q^{2n-k}$
                              . Meanwhile, the definition of $Q_{2n,n}$ yields
                              begin{align*}
                              Q_{2n,n} & =sum_{k=n}^{2n}dbinom{2n}{k}p^{k}q^{2n-k}=underbrace{sum
                              _{k=n+1}^{2n}dbinom{2n}{k}p^{k}q^{2n-k}}_{=Q_{2n,n+1}}+dbinom{2n}{n}
                              p^{n}underbrace{q^{2n-n}}_{=q^{n}}\
                              & =Q_{2n,n+1}+dbinom{2n}{n}p^{n}q^{n}.
                              end{align*}

                              Thus,
                              eqref{darij1.pf.t1.1} becomes
                              begin{align*}
                              Q_{2n+2,n+2} & =underbrace{Q_{2n,n}}_{=Q_{2n,n+1}+dbinom{2n}{n}p^{n}q^{n}
                              }-dbinom{2n}{n}p^{n}q^{n+1}-dbinom{2n}{n}p^{n+1}q^{n+1}-dbinom{2n}
                              {n+1}p^{n+1}q^{n+1}\
                              & =Q_{2n,n+1}+dbinom{2n}{n}p^{n}q^{n}-dbinom{2n}{n}p^{n}q^{n+1}-dbinom
                              {2n}{n}p^{n+1}q^{n+1}-dbinom{2n}{n+1}p^{n+1}q^{n+1}\
                              & =Q_{2n,n+1}+dbinom{2n}{n}underbrace{left( p^{n}q^{n}-p^{n}
                              q^{n+1}-p^{n+1}q^{n+1}right) }_{=left( 1-q-pqright) p^{n}q^{n}}
                              -dbinom{2n}{n+1}p^{n+1}q^{n+1}\
                              & =Q_{2n,n+1}+dbinom{2n}{n}underbrace{left( 1-q-pqright) }_{=p^{2}}
                              p^{n}q^{n}-dbinom{2n}{n+1}p^{n+1}q^{n+1}\
                              & =Q_{2n,n+1}+dbinom{2n}{n}underbrace{p^{2}p^{n}}_{=p^{n+2}}q^{n}
                              -dbinom{2n}{n+1}p^{n+1}q^{n+1}\
                              & =Q_{2n,n+1}+dbinom{2n}{n}p^{n+2}q^{n}-dbinom{2n}{n+1}p^{n+1}q^{n+1}.
                              end{align*}

                              This proves Theorem 1. $blacksquare$







                              share|cite|improve this answer












                              share|cite|improve this answer



                              share|cite|improve this answer










                              answered Jan 13 at 13:12









                              darij grinbergdarij grinberg

                              10.5k33062




                              10.5k33062























                                  0












                                  $begingroup$

                                  We have for the first sum



                                  $$S = sum_{k=0}^n {2n+2choose k+n+2} p^{k+n+2} q^{n-k}$$



                                  and for the second one



                                  $$T = sum_{k=0}^{n-1} {2nchoose k+n+1} p^{k+n+1} q^{n-1-k}$$



                                  and seek to show



                                  $$S-T =
                                  {2nchoose n} p^{n+2} q^n - {2nchoose n+1} p^{n+1} q^{n+1}$$



                                  where $p+q=1.$ We see that with



                                  $$Q_n = sum_{k=0}^n {2n+2choose k+n+2} p^{k+n+2} q^{n-k}$$



                                  the claim becomes



                                  $$Q_n - Q_{n-1} =
                                  {2nchoose n} p^{n+2} q^n - {2nchoose n+1} p^{n+1} q^{n+1}.$$



                                  Now



                                  $$Q_n = p^{n+2} q^n
                                  sum_{k=0}^n {2n+2choose k+n+2} p^{k} q^{-k}
                                  = p^{n+2} q^n
                                  sum_{k=0}^n {2n+2choose n-k} p^{k} q^{-k}
                                  \ = p^{n+2} q^n
                                  sum_{k=0}^n p^{k} q^{-k}
                                  [z^{n-k}] (1+z)^{2n+2}
                                  = p^{n+2} q^n [z^n] (1+z)^{2n+2}
                                  sum_{k=0}^n p^{k} q^{-k} z^k.$$



                                  We may extend $k$ beyond $n$ owing to the coefficient extrator $[z^n]$
                                  in front:



                                  $$p^{n+2} q^n [z^n] (1+z)^{2n+2}
                                  sum_{kge 0} p^{k} q^{-k} z^k
                                  = p^{n+2} q^n [z^n] (1+z)^{2n+2} frac{1}{1-pz/q}
                                  \ = p^2 [z^n] (1+pqz)^{2n+2} frac{1}{1-p^2z}.$$



                                  We thus have



                                  $$Q_{n-1} = p^2 [z^{n-1}] (1+pqz)^{2n} frac{1}{1-p^2z}
                                  \ = p^2 [z^n] z (1+pqz)^{2n} frac{1}{1-p^2z}.$$



                                  Subtracting we find



                                  $$Q_n - Q_{n-1}
                                  = p^2 [z^n] ((1+pqz)^2-z) (1+pqz)^{2n} frac{1}{1-p^2z}
                                  \ = p^2 [z^n] (1-p^2 z) (1-q^2 z)
                                  (1+pqz)^{2n} frac{1}{1-p^2z}
                                  \ = p^2 [z^n] (1-q^2 z) (1+pqz)^{2n}
                                  \ = p^2 [z^n] (1+pqz)^{2n}
                                  - p^2 [z^n] q^2 z (1+pqz)^{2n}
                                  \ = p^2 p^n q^n {2nchoose n}
                                  - p^2 q^2 [z^{n-1}] (1+pqz)^{2n}
                                  \ = p^{n+2} q^n {2nchoose n}
                                  - p^2 q^2 p^{n-1} q^{n-1} {2nchoose n-1}.$$



                                  This is indeed



                                  $$bbox[5px,border:2px solid #00A000]{
                                  p^{n+2} q^n {2nchoose n}
                                  - p^{n+1} q^{n+1} {2nchoose n+1}}$$



                                  as claimed.



                                  Remark. It might be simpler not to merge the $p^n q^n$ into the
                                  coefficient extractor. Further commentary TBA.






                                  share|cite|improve this answer











                                  $endgroup$


















                                    0












                                    $begingroup$

                                    We have for the first sum



                                    $$S = sum_{k=0}^n {2n+2choose k+n+2} p^{k+n+2} q^{n-k}$$



                                    and for the second one



                                    $$T = sum_{k=0}^{n-1} {2nchoose k+n+1} p^{k+n+1} q^{n-1-k}$$



                                    and seek to show



                                    $$S-T =
                                    {2nchoose n} p^{n+2} q^n - {2nchoose n+1} p^{n+1} q^{n+1}$$



                                    where $p+q=1.$ We see that with



                                    $$Q_n = sum_{k=0}^n {2n+2choose k+n+2} p^{k+n+2} q^{n-k}$$



                                    the claim becomes



                                    $$Q_n - Q_{n-1} =
                                    {2nchoose n} p^{n+2} q^n - {2nchoose n+1} p^{n+1} q^{n+1}.$$



                                    Now



                                    $$Q_n = p^{n+2} q^n
                                    sum_{k=0}^n {2n+2choose k+n+2} p^{k} q^{-k}
                                    = p^{n+2} q^n
                                    sum_{k=0}^n {2n+2choose n-k} p^{k} q^{-k}
                                    \ = p^{n+2} q^n
                                    sum_{k=0}^n p^{k} q^{-k}
                                    [z^{n-k}] (1+z)^{2n+2}
                                    = p^{n+2} q^n [z^n] (1+z)^{2n+2}
                                    sum_{k=0}^n p^{k} q^{-k} z^k.$$



                                    We may extend $k$ beyond $n$ owing to the coefficient extrator $[z^n]$
                                    in front:



                                    $$p^{n+2} q^n [z^n] (1+z)^{2n+2}
                                    sum_{kge 0} p^{k} q^{-k} z^k
                                    = p^{n+2} q^n [z^n] (1+z)^{2n+2} frac{1}{1-pz/q}
                                    \ = p^2 [z^n] (1+pqz)^{2n+2} frac{1}{1-p^2z}.$$



                                    We thus have



                                    $$Q_{n-1} = p^2 [z^{n-1}] (1+pqz)^{2n} frac{1}{1-p^2z}
                                    \ = p^2 [z^n] z (1+pqz)^{2n} frac{1}{1-p^2z}.$$



                                    Subtracting we find



                                    $$Q_n - Q_{n-1}
                                    = p^2 [z^n] ((1+pqz)^2-z) (1+pqz)^{2n} frac{1}{1-p^2z}
                                    \ = p^2 [z^n] (1-p^2 z) (1-q^2 z)
                                    (1+pqz)^{2n} frac{1}{1-p^2z}
                                    \ = p^2 [z^n] (1-q^2 z) (1+pqz)^{2n}
                                    \ = p^2 [z^n] (1+pqz)^{2n}
                                    - p^2 [z^n] q^2 z (1+pqz)^{2n}
                                    \ = p^2 p^n q^n {2nchoose n}
                                    - p^2 q^2 [z^{n-1}] (1+pqz)^{2n}
                                    \ = p^{n+2} q^n {2nchoose n}
                                    - p^2 q^2 p^{n-1} q^{n-1} {2nchoose n-1}.$$



                                    This is indeed



                                    $$bbox[5px,border:2px solid #00A000]{
                                    p^{n+2} q^n {2nchoose n}
                                    - p^{n+1} q^{n+1} {2nchoose n+1}}$$



                                    as claimed.



                                    Remark. It might be simpler not to merge the $p^n q^n$ into the
                                    coefficient extractor. Further commentary TBA.






                                    share|cite|improve this answer











                                    $endgroup$
















                                      0












                                      0








                                      0





                                      $begingroup$

                                      We have for the first sum



                                      $$S = sum_{k=0}^n {2n+2choose k+n+2} p^{k+n+2} q^{n-k}$$



                                      and for the second one



                                      $$T = sum_{k=0}^{n-1} {2nchoose k+n+1} p^{k+n+1} q^{n-1-k}$$



                                      and seek to show



                                      $$S-T =
                                      {2nchoose n} p^{n+2} q^n - {2nchoose n+1} p^{n+1} q^{n+1}$$



                                      where $p+q=1.$ We see that with



                                      $$Q_n = sum_{k=0}^n {2n+2choose k+n+2} p^{k+n+2} q^{n-k}$$



                                      the claim becomes



                                      $$Q_n - Q_{n-1} =
                                      {2nchoose n} p^{n+2} q^n - {2nchoose n+1} p^{n+1} q^{n+1}.$$



                                      Now



                                      $$Q_n = p^{n+2} q^n
                                      sum_{k=0}^n {2n+2choose k+n+2} p^{k} q^{-k}
                                      = p^{n+2} q^n
                                      sum_{k=0}^n {2n+2choose n-k} p^{k} q^{-k}
                                      \ = p^{n+2} q^n
                                      sum_{k=0}^n p^{k} q^{-k}
                                      [z^{n-k}] (1+z)^{2n+2}
                                      = p^{n+2} q^n [z^n] (1+z)^{2n+2}
                                      sum_{k=0}^n p^{k} q^{-k} z^k.$$



                                      We may extend $k$ beyond $n$ owing to the coefficient extrator $[z^n]$
                                      in front:



                                      $$p^{n+2} q^n [z^n] (1+z)^{2n+2}
                                      sum_{kge 0} p^{k} q^{-k} z^k
                                      = p^{n+2} q^n [z^n] (1+z)^{2n+2} frac{1}{1-pz/q}
                                      \ = p^2 [z^n] (1+pqz)^{2n+2} frac{1}{1-p^2z}.$$



                                      We thus have



                                      $$Q_{n-1} = p^2 [z^{n-1}] (1+pqz)^{2n} frac{1}{1-p^2z}
                                      \ = p^2 [z^n] z (1+pqz)^{2n} frac{1}{1-p^2z}.$$



                                      Subtracting we find



                                      $$Q_n - Q_{n-1}
                                      = p^2 [z^n] ((1+pqz)^2-z) (1+pqz)^{2n} frac{1}{1-p^2z}
                                      \ = p^2 [z^n] (1-p^2 z) (1-q^2 z)
                                      (1+pqz)^{2n} frac{1}{1-p^2z}
                                      \ = p^2 [z^n] (1-q^2 z) (1+pqz)^{2n}
                                      \ = p^2 [z^n] (1+pqz)^{2n}
                                      - p^2 [z^n] q^2 z (1+pqz)^{2n}
                                      \ = p^2 p^n q^n {2nchoose n}
                                      - p^2 q^2 [z^{n-1}] (1+pqz)^{2n}
                                      \ = p^{n+2} q^n {2nchoose n}
                                      - p^2 q^2 p^{n-1} q^{n-1} {2nchoose n-1}.$$



                                      This is indeed



                                      $$bbox[5px,border:2px solid #00A000]{
                                      p^{n+2} q^n {2nchoose n}
                                      - p^{n+1} q^{n+1} {2nchoose n+1}}$$



                                      as claimed.



                                      Remark. It might be simpler not to merge the $p^n q^n$ into the
                                      coefficient extractor. Further commentary TBA.






                                      share|cite|improve this answer











                                      $endgroup$



                                      We have for the first sum



                                      $$S = sum_{k=0}^n {2n+2choose k+n+2} p^{k+n+2} q^{n-k}$$



                                      and for the second one



                                      $$T = sum_{k=0}^{n-1} {2nchoose k+n+1} p^{k+n+1} q^{n-1-k}$$



                                      and seek to show



                                      $$S-T =
                                      {2nchoose n} p^{n+2} q^n - {2nchoose n+1} p^{n+1} q^{n+1}$$



                                      where $p+q=1.$ We see that with



                                      $$Q_n = sum_{k=0}^n {2n+2choose k+n+2} p^{k+n+2} q^{n-k}$$



                                      the claim becomes



                                      $$Q_n - Q_{n-1} =
                                      {2nchoose n} p^{n+2} q^n - {2nchoose n+1} p^{n+1} q^{n+1}.$$



                                      Now



                                      $$Q_n = p^{n+2} q^n
                                      sum_{k=0}^n {2n+2choose k+n+2} p^{k} q^{-k}
                                      = p^{n+2} q^n
                                      sum_{k=0}^n {2n+2choose n-k} p^{k} q^{-k}
                                      \ = p^{n+2} q^n
                                      sum_{k=0}^n p^{k} q^{-k}
                                      [z^{n-k}] (1+z)^{2n+2}
                                      = p^{n+2} q^n [z^n] (1+z)^{2n+2}
                                      sum_{k=0}^n p^{k} q^{-k} z^k.$$



                                      We may extend $k$ beyond $n$ owing to the coefficient extrator $[z^n]$
                                      in front:



                                      $$p^{n+2} q^n [z^n] (1+z)^{2n+2}
                                      sum_{kge 0} p^{k} q^{-k} z^k
                                      = p^{n+2} q^n [z^n] (1+z)^{2n+2} frac{1}{1-pz/q}
                                      \ = p^2 [z^n] (1+pqz)^{2n+2} frac{1}{1-p^2z}.$$



                                      We thus have



                                      $$Q_{n-1} = p^2 [z^{n-1}] (1+pqz)^{2n} frac{1}{1-p^2z}
                                      \ = p^2 [z^n] z (1+pqz)^{2n} frac{1}{1-p^2z}.$$



                                      Subtracting we find



                                      $$Q_n - Q_{n-1}
                                      = p^2 [z^n] ((1+pqz)^2-z) (1+pqz)^{2n} frac{1}{1-p^2z}
                                      \ = p^2 [z^n] (1-p^2 z) (1-q^2 z)
                                      (1+pqz)^{2n} frac{1}{1-p^2z}
                                      \ = p^2 [z^n] (1-q^2 z) (1+pqz)^{2n}
                                      \ = p^2 [z^n] (1+pqz)^{2n}
                                      - p^2 [z^n] q^2 z (1+pqz)^{2n}
                                      \ = p^2 p^n q^n {2nchoose n}
                                      - p^2 q^2 [z^{n-1}] (1+pqz)^{2n}
                                      \ = p^{n+2} q^n {2nchoose n}
                                      - p^2 q^2 p^{n-1} q^{n-1} {2nchoose n-1}.$$



                                      This is indeed



                                      $$bbox[5px,border:2px solid #00A000]{
                                      p^{n+2} q^n {2nchoose n}
                                      - p^{n+1} q^{n+1} {2nchoose n+1}}$$



                                      as claimed.



                                      Remark. It might be simpler not to merge the $p^n q^n$ into the
                                      coefficient extractor. Further commentary TBA.







                                      share|cite|improve this answer














                                      share|cite|improve this answer



                                      share|cite|improve this answer








                                      edited Jan 13 at 16:11

























                                      answered Jan 13 at 15:55









                                      Marko RiedelMarko Riedel

                                      39.7k339108




                                      39.7k339108






























                                          draft saved

                                          draft discarded




















































                                          Thanks for contributing an answer to Mathematics Stack Exchange!


                                          • Please be sure to answer the question. Provide details and share your research!

                                          But avoid



                                          • Asking for help, clarification, or responding to other answers.

                                          • Making statements based on opinion; back them up with references or personal experience.


                                          Use MathJax to format equations. MathJax reference.


                                          To learn more, see our tips on writing great answers.




                                          draft saved


                                          draft discarded














                                          StackExchange.ready(
                                          function () {
                                          StackExchange.openid.initPostLogin('.new-post-login', 'https%3a%2f%2fmath.stackexchange.com%2fquestions%2f3071705%2fcounting-proability-and-binomial-coefficients%23new-answer', 'question_page');
                                          }
                                          );

                                          Post as a guest















                                          Required, but never shown





















































                                          Required, but never shown














                                          Required, but never shown












                                          Required, but never shown







                                          Required, but never shown

































                                          Required, but never shown














                                          Required, but never shown












                                          Required, but never shown







                                          Required, but never shown







                                          Popular posts from this blog

                                          Mario Kart Wii

                                          What does “Dominus providebit” mean?

                                          Antonio Litta Visconti Arese